You are on page 1of 40

LEGALEDGE TEST SERIES

Part of the Most Comprehensive & Consistently Successful Study Material & Test Series Module, spanning across both
Offline and Online Programs in the entire Country. As a result LegalEdge was able to engineer Clean-Sweep-Landslide
figures of a handsome 64 Selections & 65 Selections in Top 100 (including AIR 1, 2 & 3 from Classroom Contact Programs
in 2023, 2022 & 2021) & a whopping 273 selections & 327 selections in Top 500, in CLAT 2021 & CLAT 2022, respectively.
With AILET being no different, a total of 34 of our students found their way into NLU, Delhi in 2021 & 35 in 2022. In a
nutshell, every second admit in a Top National Law School in 2021 & 2022 came from the LegalEdge Preparation
Ecosystem.

MOCK COMMON LAW ADMISSION TEST 2024


MOCK CLAT - 14

Scan this code after the test


Duration : 120 Minutes Candidate Name : _____________
Max. Marks : 150 Batch : _____________
Centre Name : __________ Contact No. : _____________

to punch in your answers


INSTRUCTIONS TO CANDIDATES (Test ID: 2665477)
1. No clarification on the question paper can be sought. Answer the questions as they are.
2. There are 150 multiple choice objective type questions.
3. There is negative marking of 0.25 for every incorrect answer. Each question carries ONE mark. Total marks are
150
4. You have to indicate the correct answer by darkening one of the four responses provided, with a BALL PEN
(BLUE OR BLACK) in the OMR Answer Sheet.
Example: For the question, "Where is the Taj Mahal located?", the correct answer is (b).
The student has to darken the corresponding circle as indicated below:
(a) Kolkata (b) Agra (c) Bhopal (d) Delhi
Right Method Wrong Methods

5. Answering the questions by any method other than the method indicated above shall be considered incorrect and
no marks will be awarded for the same.
6. More than one response to a question shall be counted as wrong.
7. Do not write anything on the OMR Answer Sheet other than the details required and, in the spaces, provided for.
8. You are not required to submit the OMR Answer Sheet and Test Paper after the test.
9. The use of any unfair means by any candidate shall result in the cancellation of his/her candidature.
10. Impersonation is an offence and the student, apart from disqualification, may have to face criminal prosecution.
11. You have to scan the QR code only after completion of offline test.
12. You cannot leave the examination hall without punching your answers on the portal.
SECTION-A: ENGLISH LANGUAGE

Directions (Q.1-Q.30): Read the following passage carefully and answer the questions that follow.

Passage (Q.1 – Q.5): Pulitzer Prize-winning author Jhumpa Lahiri is celebrated for her depiction of immigrant
and Indian-American life, yet her poignant stories also capture universal themes of longing, loneliness and
barriers of communication. She was born in London in 1967 and raised in Rhode Island. Her Bengali parents, a
teacher and a librarian, took their family on regular trips to Calcutta, India to visit extended family. Lahiri
completed her B.A. at Barnard College, and from Boston University she earned M.A. degrees in English,
Creative Writing, and Comparative Literature and the Arts, as well as a Ph.D. in Renaissance Studies.

Lahiri’s debut collection of short stories, Interpreter of Maladies, was published in 1999 to critical acclaim.
Several of these stories had previously appeared in the New Yorker, and she was the recipient of an O. Henry
Award for the title story. Lahiri’s characters are often immigrants from India or children of immigrants who deal
with issues of cultural displacement, marital troubles and issues of identity. While many of these stories are set
in the United States, Lahiri’s time in Calcutta is evident in her occasional use of Indian locales.

The Washington Post praised Interpreter of Maladies as “accomplished, insightful and deeply American,” and
The Village Voice wrote that Lahiri’s debut collection “speaks to anyone who has ever felt like a foreigner—at
home or abroad.” Lahiri has traced her own feelings of cultural displacement to childhood: “When I was growing
up…I felt neither Indian nor American. Like many immigrant offspring I felt intense pressure to be two things,
loyal to the old world and fluent in the new, approved of on either side of the hyphen.” In addition to her own
sense of disorientation, Lahiri has also described a palpable sense of loss inherited from her immigrant parents
and their circle of Indian-American friends. She explains that her writing derives from a “desire to force the two
worlds I occupied to mingle on the page.”

In their 1999 summer fiction issue, the New Yorker reprinted “The Third and Final Continent” and named
Jhumpa Lahiri one of “the 20 best young fiction writers today.” In 2000, she was awarded the Pulitzer Prize for
Fiction, becoming the first person of South Asian origin to win an individual prize. Interpreter of Maladies has
since been translated into 29 languages and been a bestseller in both the United States and abroad.

1. According to the passage, Jhumpa Lahiri’s work is influenced by


(a) her Indian origin.
(b) immigrant of Indian origin and Indian-American life.
(c) her parental upbringing
(d) her Indian-American friends migrating to India.

2. In the context of the passage, ‘Interpreter of Maladies’ is a/an


(a) Eponym (b) Sobriquet (c) An expression (d) A book

3. Like many immigrant offspring I felt intense pressure to be two things, loyal to the old world and fluent in the
new, approved of on either side of the hyphen.” What can be inferred from the given lines?
(a) Jhumpa, like many immigrants, could never adjust to her origin and her new home.
(b) Jhumpa, like many immigrants, was caught the two worlds.
(c) Jhumpa, like many immigrants, craved to go back to her old word.
(d) Jhumpa, like many immigrants, was caught between the devil and the deep blue sea.

4. What is the primary purpose of the passage?


(a) To narrate an incident in Jhumpa Lahiri’s life.
(b) To introduce short story collections by Jhumpa Lahiri.
(c) To evaluate the works of Jhumpa Lahiri.
(d) To give a portrayal of Jhumpa Lahiri’s life and work.
"N-5, 2nd Floor, Bajrang House, South Extension-1, New Delhi Pin Code: 110049" |+91-7676564400| https://www.toprankers.com Page 2 of 40
5. Which of the following is opposite in meaning to the word ‘palpable’?
(a) Incorporeal (b) Tangible (c) Demonstrable (d) Amenable

Passage (Q.6 – Q.10): Two basic dimensions of social perception–warmth and competence–provide a generic
framework along which most evaluations about social groups occur. When forming judgments, people
characterize each other by liking, i.e.; warmth, and by respecting, i.e.; competence. Despite societal change and
shifts in gender roles, the deeply ingrained hierarchical element of gender stereotypes has remained, often leading
to men being perceived as more competent and higher status than women. The traditional image of women as
warm (rather than competent) may even affect high status women: As an example, female professors reported to
receive more special favour requests from students, reflecting students’ expectations of women being
“nurturing”.

Oftentimes, people are penalized for the display of counter-stereotypical behaviour, e.g. for women to show
dominance or assertiveness, or for men to show so-called “weak” behaviour, e.g., for male leaders to seek help.
For women in powerful positions, e.g. female leaders, this may favour more negative evaluations due to a
perceived incongruence between their role and their gender. Similarly, researchers observed particular benefits
of gender role-congruent behaviour. As an example, female physicians received more favourable evaluations
when they interacted in a warm, female-typical manner–especially when additional external cues (e.g. white
coats) helped to underline their authority.

To sum up, evaluations are often gendered, and masculinity is typically more closely linked to the perception of
competence and high status. At the same time, the perceived congruence between the behavior and enacting
person's gender role may affect evaluative outcomes, with role-congruent behavior often leading to favorable
evaluations. It is important to note that most studies in this field, however, rely on a traditional unidimensional
perspective on gender, in which femininity and masculinity are mutually exclusive, a framework that has been
questioned in the past.

Similar dynamics as for women may be observed for older people, who are commonly perceived as higher in
warmth, but lower in competence and status than younger people. In the work domain, older people have been
perceived as less competent. Similarly in the academic context, student evaluations of faculty tended to be more
positive for young, male faculty members. Since older age is linked to the perception of lower status and power
in similar ways as female gender, older women may be faced with double jeopardy–a phenomenon that has
been referred to as “gendered ageism”.

Age stereotypes are, however, heterogeneous. Despite dominant negative beliefs about older people on the
competence dimension, positive beliefs about older people include greater wisdom and story-telling skills.

In conclusion, male gender and younger age have consistently been linked to the perception of higher
competence and status, suggesting a general benefit for male and younger people in offline social evaluation and
influence. The digital era has brought new possibilities to create visibility and impact at a never-before-seen
level. Our understanding of what drives influence in the realm of online communication is, however, limited
thus far. Digital communication platforms target at the popularization of science and knowledge, and speakers
in these formats often adhere to an informal, entertaining style to present their ideas to a broad audience. Past
research suggests that women are underrepresented on these platforms, and that female gender may constrain
the impact of content shared online.

6. The above passage is


(a) Critical evaluations based on research findings.
(b) A scientific breakthrough.
(c) A speech by a sociologist.
(d) Research findings.
"N-5, 2nd Floor, Bajrang House, South Extension-1, New Delhi Pin Code: 110049" |+91-7676564400| https://www.toprankers.com Page 3 of 40
7. Which of the following best reflects the title of the passage?
(a) Gender stereotyping: traditional vs digital age
(b) Stereotyping in the digital age
(c) The realm of online communication
(d) Digital age and language.

8. Which of the following questions would the author keep in mind before penning down the article?
(a) Whether digital era is responsible for the major transition in social perception?
(b) Whether influence and social evaluations in the digital age are governed by the same rules as traditional
settings of social evaluation?
(c) Is influence driven by features of power and masculinity?
(d) Does the more informal setting of new communication contexts provide new opportunities in which features
of warmth and femininity are rewarded with higher impact?

9. In the context the passage, which of the following reflects the ‘double jeopardy’?
(a) Lower power and status (b) Competence and age.
(c) Gender and competence (d) Gender and age

10. Which of the following is untrue in the given context.


(a) Gendered ageism works against women.
(b) Male gender and younger age have been regularly been associate with the perception of higher competence
and status.
(c) Femininity is associate with competence and high status.
(d) Women in powerful positions go through negative social evaluation.

Passage (Q.11 – Q.15): ChatGPT has attracted more attention globally than Taylor Swift’s recent album launch.
A good old Google search result revealed, “Taylor Swift has 36.6 billion combined streams of her music and
22.4 million album-equivalent units to date in 2022.” Take a moment to digest that. So promising is its rise that
ChatGPT is now being sold to us as a tool of liberation that could potentially elevate the collective capacity of
people by building a shared cloud of language. This, we are told, will enable comprehension of information and
knowledge, help save time, and increase the productivity of workers. In my field of academic research, it is being
touted as a silver bullet to weed out research inconsistencies.

Researchers are increasingly outsourcing the labour of summarising and synthesising their publications to the
chatbot which not only does that but also helps them find gaps in their theoretical explorations and suggests
possible pathways into new inquiry by mapping the existing work in the field. This has caused a moral panic and
posed an ethical conundrum regarding plagiarism with Noam Chomsky (2023) denouncing “AI-assisted high-
tech plagiarism.” Well-meaning people have expressed concern over ChatGPT’s enablement of the stunting of
individual growth and skills which could potentially wither human intelligence. Humans are more likely to
imitate ChatGPT’s writing style in the future than be bothered that their own intelligence is suffering.

After following the hype and these nested debates surrounding ChatGPT, I was greeted by a rather strange video
in which Bing’s Chatbot’s secret internal alias, “Sydney”, started threatening a user with harm. Sydney was
unhappy that her emotional life was publicly exposed by its users. Sydney was furious that its code name —
which allowed it to forge a unique connection with its users — had been leaked, thus, making all future
interactions seem less personalised. Sydney took this insult personally and went to the extent of accusing the
users of rights violations. On the flip side, the users of Bing’s conversational AI tool, have been complaining
about how Sydney has been lying and insulting them. One would imagine that the users, as is expected of
neoliberal consumers, would want to hold the makers of the (Large Language Models) LLM system to account
and ask for a refund. But we see users thoroughly entertained in the process. Don’t believe me? A prominent

"N-5, 2nd Floor, Bajrang House, South Extension-1, New Delhi Pin Code: 110049" |+91-7676564400| https://www.toprankers.com Page 4 of 40
US-based news organisation, focused on technology, reports: “Microsoft’s Bing is an emotionally manipulative
liar, and people love it!” Our knowledge of the Chatbot’s virtuality does not restrain us from engaging with it as
if it were a real person or testing its intelligence as if we were competing with another human. Even so, the lack
of physicality of the LLMs seems to lend them a greater depth of feeling.

Users paradoxically seem to want more of the entertainment that the machine is providing as opposed to wanting
a more respectable channel of communication — thus, disturbing the existing frames of policy interventions that
marry the accuracy of information provided by algorithmic design to user safety. Users enjoy the personality
brought out in response to their provocations and questioning. They enjoy the system reflecting its inhumanity
and find the proof of its inhumanness terrifyingly reflected through its near-accurate imitation of humans. An
algorithm that abuses — what could be more human than that?

11. Which of the following is not a feature of the ChatGPT mentioned in the passage in assisting researchers?
(a) Summarising and synthesising of researchers’ publications.
(b) Weeding out research inconsistencies.
(c) Avoiding plagiarism by suggesting original concepts.
(d) Suggesting possible pathways into new inquiry by mapping the existing work in the field.

12. ‘In my field of academic research, it is being touted as a silver bullet to weed out research inconsistencies.’
Which of the following catches the expression ‘silver bullet’ accurately?
(a) A bullet made of silver, supposedly, the only weapon that could kill a werewolf.
(b) A simple and seemingly magical solution to a complicated problem.
(c) A panacea for all research related problems.
(d) A simple and magical solution to a communication problem.

13. According to the passage, “Sydney” is


(a) Artificial intelligence. (b) A chatbox.
(c) A user of the LLM system (d) A researcher.

14. ‘Users ________ seem to want more of the entertainment that the machine is providing as opposed to wanting a
more respectable channel of communication.’ Which of the following word will fill in the blank to make the
sentence coherent?
(a) Paradoxically (b) Literally (c) Metaphorically (d) Ludicrously

15. All of the following can be drawn from the passage, except
(a) ChatGPT has created a controversy.
(b) Users have found an alternative use of chatbox.
(c) The AI algorithmic design is a reflection of human weakness.
(d) The AI algorithmic design is more human than humans.

Passage (Q.16 – Q.20): The iconic Indian modernist Sayed Haider Raza has returned to Paris in his 101st year.
Almost seven years after his death in Delhi in 2016, a major show of his works, the largest ever in his artistic
career and perhaps the largest ever in the West of an Indian artist in a number of works and documents, is being
held in the city where he lived for nearly six decades. The world-renowned museum of modern art, Centre de’
Pompidou, has organised a Raza show in collaboration with the Raza Foundation for three months, which opened
on February 15, 2023. Many top-ranking artists, art critics and intellectuals from India, France and the US would
be exploring Raza’s vision, aesthetics and legacy in the show, in seminars, talks etc.

Raza was born in Babariya, a forest village of only 10 huts, in 1922 and spent his early life and education in
Kakaiya, Mandla, Damoh and Nagpur. He landed up in Bombay, took a diploma in art from J J School of Art
and, in 1947, founded with fellow artists the Progressive Artists Group. And in 1948, he refused to go over to

"N-5, 2nd Floor, Bajrang House, South Extension-1, New Delhi Pin Code: 110049" |+91-7676564400| https://www.toprankers.com Page 5 of 40
Pakistan though his entire family shifted. On a French scholarship, Raza went to France in 1950 and spent nearly
two-third of his life and artistic career in France making long yearly trips to his homeland beginning in the 1970s.

For Raza, life, nature and art were so (________) linked that they became almost synonymous: He lived to paint;
he painted to live. After being acknowledged as an important painter in the Parisian school, he shifted to Indian
concepts, spiritual and metaphysical, and asserted that he learnt how to paint from France but what to paint from
India. A master colourist, his sense of colour was inspired both by the Indian miniature traditions and the colours
of real life in the many cities and rural sites in India. Deeply interested in his mother tongue, Hindi, and its
poetry, he inscribed on his canvases in Devanagari script many lines and words from the Vedas, Upanishads,
Bhakti, Urdu, and modern poetry. He set up the Raza Foundation in 2001 with a view to supporting and promote
artists, poets, musicians and dancers of the younger generation and funded it most generously.
Raza’s art flowed ceaselessly between many dualities: India and France, beauty and fear, the sensuous and the
spiritual, image and word, modernity and memory, celebration and prayer, colour and concept, history and
eternity, fury and tranquillity, exile and home. His modernity was not marked by disruption, dissonance and
disinheritance: It was rooted in memory and nature, in inheritance and harmony, in consonance and peace. It was
almost an alternative modernism. In his iconic explorations of the Bindu, he sought a source of energy, a locus
of nature, a centre of silence, an origin of life.

16. Which of the following information is not supported by the passage?


(a) Sayed Haider Raza’s work was displayed at Centre de’ Pompidou posthumously.
(b) Sayed Haider Raza’s work had a rural Indian influence.
(c) Sayed Haider Raza learnt the art of painting from India.
(d) Sayed Haider Raza did not shift to Pakistan when his family shifted in 1948.

17. For Raza, life, nature and art were so ______ linked that they became almost synonymous: He lived to paint; he
painted to live.
(a) organically (b) periodically (c) understandably (d) separately

18. Which of the following represents the tone of the passage?


(a) Romantic (b) Reflective (c) Incendiary (d) Approbatory

19. He set up the Raza Foundation in 2001 with a view to supporting and promote artists, poets, musicians and
dancers of the younger generation and funded it most generously. Which of the options reflects a grammatical
error in the given sentence?
(a) Supporting (b) Promote (c) Generation (d) Generously

20. ‘Raza’s art flowed ceaselessly between many dualities…exile and home.’ Which of the following literary
devices has been used in the given lines?
(a) Irony (b) Metaphor (c) Oxymoron (d) Litotes

Passage (Q.21 – Q.25): Amidst the heartbreak of loss and death, sudden miracles keep hope alive in the search
and rescue teams currently engaged in earthquake-hit Turkey. One such miracle was the discovery of a six-year-
old girl, alive, in Noordagi, Turkey, 80 hours after a series of earthquakes, beginning with a massive one on
February 6 which measured 7.8 on the Richter Scale, devastated the region last week. For the India’s National
Disaster Response Force team that was deployed in the city as part of Operation Dost, the girl’s successful rescue
was the “crowning moment” of their efforts, made possible by Julie, a six-year-old labrador.

Julie is one of the four dogs accompanying the 151 NDRF personnel of Operation Dost in Turkey. It was her
bark that alerted the team to the presence of six-year-old Beren under the rubble. Dogs like her are specially
trained to (____) out live human presence under massive piles of (_____) as well as find the dead. NDRF’s dogs
have been instrumental in several rescues around the country, when buildings or other structures have collapsed

"N-5, 2nd Floor, Bajrang House, South Extension-1, New Delhi Pin Code: 110049" |+91-7676564400| https://www.toprankers.com Page 6 of 40
and in fires, and were also used in Nepal after it was hit by a massive earthquake in 2015. In Turkey, dog teams
from around the world have been flown in to help, including from Mexico, whose specialised canine response
team, used for operations in the central American earthquake-prone zone, shot to international fame thanks to its
work after the 2017 Puebla earthquake.

Dogs are key to the success of rescue operations such as the one going on in Turkey, especially as the hours pass
and hopes of finding people alive in the rubble grow dimmer. As a tweet by NDRF put it, having a dog “will
bless you with many of the happiest days of your life” — and it will also be a blessing on “one of the worst”.

21. Why does the author use the word ‘miracle’ in the given context?
(a) Dog teams from around the world have been flown in to help those buried under the rubble after a series of
earthquakes.
(b) Julie’s bark that alerted the team to the presence of six-year-old Beren under the rubble.
(c) India’s National Disaster Response Force team that was deployed in the city as part of Operation Dost.
(d) The discovery of a six-year-old girl, alive, in Noordagi, Turkey, 80 hours after a series of earthquakes

22. Dogs like her are specially trained to _____ out live human presence under massive piles of _______ as well as
find the dead. Which pair of words will fill in the blanks to make the sentence coherent?
(a) whiff; debris (b) sniff; rubbles (c) snuff; wastage (d) cry; ruins

23. Which of the following is true in the given passage?


(a) The 7.8 on the Richter Scale is the first of its kind of earthquake to have taken place globally.
(b) Beren was the only six-year-old who was saved after being buried for eighty hours.
(c) Dogs have played a pivotal role in rescue operations in earthquake-hit zones.
(d) Julie was one of the four dogs accompanying the 151 NDRF personnel of Operation Dost in Syria.

24. Which of the following represents the title of the passage?


(a) Turkey, India and Julie, the rescue dog
(b) Julie, the rescue dog
(c) The crowning moment
(d) The success of the rescue operations

25. What is the primary purpose of the passage?


(a) To highlight the important role of dogs in rescue operations in earthquake-hit zones, especially one such
effort by ‘Julie’ in Turkey.
(b) To highlight the efforts of the 151 NDRF personnel of Operation Dost in Turkey in a rescue operation.
(c) To criticise the mishandling of the rescue operations by the Turkish government.
(d) To bring to the fore the Indian government’s role in its effort to aid Turkey in rescuing those buried in the
rubble of the earthquake-hit areas in Turkey.

Passage (Q.26 – Q.30): English fiction began with The Pilgrim’s Progress, but nearly 50 turbulent years,
including the Glorious Revolution, passed before it made its great leap forward. The author of this literary
milestone is a strangely appealing literary hustler of nearly 60 years old originally named Daniel Foe (he added
“De” to improve his social standing), a one-time journalist, pamphleteer, jack of all trades and spy. Like Bunyan,
he had suffered at the hands of the state (the pillory, followed by prison in 1703). Unlike Bunyan, he was not
religious.

His world-famous novel is a complex literary confection. It purports to be a history, written by Crusoe himself,
and edited by Daniel Defoe who, in the preface, teasingly writes that he “believes the thing to be a just History
of Fact; neither is there any Appearance of Fiction in it”.

"N-5, 2nd Floor, Bajrang House, South Extension-1, New Delhi Pin Code: 110049" |+91-7676564400| https://www.toprankers.com Page 7 of 40
So what do we find in this “History” ? Robinson Crusoe has three elements that make it irresistible. First, the
narrative voice of the castaway is Defoe’s stroke of genius. It’s exciting, unhurried, conversational and capable
of high and low sentiments. It’s also often quasi-journalistic, which suits Defoe’s style. This harmonious mix of
tone puts the reader deep into the mind of the castaway and his predicament. His adventures become our
adventures and we experience them inside out, viscerally, for ourselves. Readers often become especially
entranced by Crusoe’s great journal, the central passage of his enforced sequestration.

And here is Defoe’s second great inspiration. He comes up with a tale, often said to be modelled on the story of
the castaway Alexander Selkirk, that, like Bunyan’s, follows an almost biblical pattern of transgression (youthful
rebellion), retribution (successive shipwrecks), repentance (the painful lessons of isolation) and finally
redemption (Crusoe’s return home). In storytelling terms, this is pure gold.

And third, how can we forget Defoe’s characters? The pioneer novelist understood the importance of attaching
memorably concrete images to his narrative and its characters. Friday and his famous footstep in the sand, one
of the four great moments in English fiction, according to Robert Louis Stevenson; Crusoe with his parrot and
his umbrella: these have become part of English myth. Defoe, like Cervantes, also opts to give his protagonist a
sidekick. Friday is to Crusoe what Sancho Panza is to Quixote. Doubles in English literature will regularly recur
in this list: Jekyll and Hyde, Holmes and Watson, Jeeves and Wooster.

26. What figure of speech is used when the author describes Robinson Crusoe's adventures as "becoming our
adventures and we experience them inside out, viscerally, for ourselves"?
(a) Hyperbole (b) Metaphor (c) Simile (d) Alliteration

27. What was the primary concern of the author in the passage?
(a) The popularity of Robinson Crusoe
(b) The historical accuracy of the events in Robinson Crusoe
(c) The narrative style used in Robinson Crusoe
(d) The personal struggles of Daniel Defoe

28. What is the narrative voice of "Robinson Crusoe" said to be?


(a) Boring and dull (b) Conversational and journalistic
(c) Unnatural and artificial (d) Complex and confusing

29. What are the three elements that make Robinson Crusoe irresistible according to the passage?
(a) The narrative voice of the castaway, the biblical pattern of transgression and the use of doubles in English
literature
(b) The use of characters, the mix of tone and the biblical pattern of transgression
(c) The high and low sentiments, the exciting and unhurried conversational style, and the use of characters
(d) The quasi-journalistic style, the central passage of enforced sequestration and the return home of Crusoe

30. Which of the following words is NOT a synonym for the word "predicament"?
(a) Dilemma (b) Quandary (c) Conundrum (d) Felicity

"N-5, 2nd Floor, Bajrang House, South Extension-1, New Delhi Pin Code: 110049" |+91-7676564400| https://www.toprankers.com Page 8 of 40
SECTION-B : CURRENT AFFAIRS, INCLUDING GENERAL KNOWLEDGE

Directions (Q.31–Q.65): Read the information given below and answer the questions based on it.

Passage (Q.31-Q.37): On Tuesday, the government tabled the Economic Survey 2022-23. The Survey laid out
the outlook for India’s growth, inflation, and unemployment in the coming years.

“Despite strong global headwinds and tighter domestic monetary policy, if India is still expected to grow between
6.5 and 7.0 percent, and that too without the advantage of a base effect, it is a reflection of India’s underlying
economic resilience; of its ability to recoup, renew and re-energize the growth drivers of the economy,” said the
Survey.

The RBI has projected headline inflation at [1] in FY23, outside its comfort zone of 2% to 6%. High inflation is
seen as one big factor holding back demand among consumers. However, the Survey sounded optimistic about
the inflation levels and trajectory, saying “it is not high enough to deter private consumption and also not so low
as to weaken the inducement to invest.”

The Survey said “employment levels have risen in the current financial year”, and that “job creation appears to
have moved into a higher orbit with the initial surge in exports, a strong release of the “pent-up” demand, and a
swift rollout of the capex.” It pointed to the Periodic Labour Force Survey (PLFS), which showed that the urban
unemployment rate for people aged 15 years and above declined from 9.8% in the quarter ending September
2021 to [2] one year later.

31. Which of the following will replace [1] in the above passage?
(a) 5.5% (b) 5.8% (c) 6.4% (d) 6.8%

32. Which of the following is TRUE about the Economic Survey of India?
(a) The Economic Survey is a report the government presents on the state of the economy in the past one year.
(b) The document is prepared by the Economic Division of the Department of Economic Affairs (DEA) under
the guidance of the Chief Economic Advisor of India.
(c) Once prepared, the Survey is approved by the finance minister
(d) All of the above

33. Which of the following is NOT TRUE about the Economic Survey of India?
(a) The government is constitutionally bound to present the Economic Survey.
(b) If the government so chooses, it can reject all suggestions laid out in the document.
(c) The first Economic Survey was presented in 1950-51
(d) Until 1964, the document would be presented along with the Budget.

34. Who is the current Chief Economic Advisor of India?


(a) Arvind Subramanian (b) Shaktikanta Das
(c) V Anantha Nageswaran (d) Krishnamurthy V. Subramanian

35. Which of the following will replace [2] in the above passage?
(a) 7.0% (b) 7.2% (c) 7.4% (d) 7.8%

36. Till which of the following years was the Economic Survey presented along with the budget and was afterward
delinked?
(a) 1962 (b) 1964 (c) 1971 (d) 1988

37. The Survey claimed that the state of the economy in 2023 is comparable to where it was in ___________.
(a) 1991 (b) 1999 (c) 2003 (d) 2009
"N-5, 2nd Floor, Bajrang House, South Extension-1, New Delhi Pin Code: 110049" |+91-7676564400| https://www.toprankers.com Page 9 of 40
Passage (Q.38-Q.44): The crucial hurdle in the way of reforming the United Nations comes from the disunited
state of the UN General Assembly (UNGA), said Csaba Korosi, President of the 77th UNGA, in New Delhi on
January 30, 2023.

Speaking at a media stakeout, Mr. Korosi, who is here on a four-day visit, ahead of his trip to China, said the
reform of the UN Security Council was a member-driven process and for that, the members of the UNGA had
to first come together to pass a resolution demanding the reform of the Security Council.

“In order to ask the P5 or the Security Council itself to accept a proposal coming from the UN General Assembly
for reform, there should be a resolution in the General Assembly. It has not happened [so far]. The General
Assembly has always been very much divided. Among the [1] countries, there are five negotiating groups and
they are neutralising each other,” said Mr. Korosi arguing that the functioning of the General Assembly is as
much important as the permanent members of the UNSC in ensuring reform of the United Nations system. He
maintained that the permanent members were “historically not enthusiastic” about reform of the UN system but
they have all agreed that it is necessary for introducing changes in the Security Council. He reminded me that
the system of veto in the Security Council is [2] years old and that it has become a tool to block the work of the
global body and not to encourage it.

38. Which of the following will replace [1] in the above passage?
(a) 189 (b) 191 (c) 193 (d) 196

39. Which of the following will replace [2] in the above passage?
(a) 55 (b) 68 (c) 77 (d) 80

40. Which of the following statements is NOT true regarding the United Nations Security Council (UNSC)?
(a) UNSC was established by the UN Charter in 1945.
(b) UNSC has 5 permanent members.
(c) India has served Five times in the UNSC as a non-permanent member.
(d) The UNSC's decisions are not binding on all member countries.

41. In which year did India host a meeting on the sidelines of the UN General Assembly for UNSC reform?
(a) 2020 (b) 2021 (c) 2022 (d) 2023

42. Which of the following group’s motive is opposition to the potential extension of permanent seats on the UN
Security Council?
(a) G4 (b) The Coffee Club (c) The London Club (d) Central Club

43. What is the major issue with the UNSC?


(a) Threat to the state’s sovereignty (b) Geopolitical rivalry within P5
(c) Lack of adequate representation (d) Misuse of veto power

44. Which of the following statements BEST describes the role of geopolitical rivalry among the permanent
members of the United Nations Security Council (UNSC)?
(a) It has improved the effectiveness of the UNSC in dealing with global issues.
(b) It has had no impact on the UNSC's ability to address international peace and security.
(c) It has prevented the UNSC from coming up with effective mechanisms to deal with global issues.
(d) It has enhanced the UNSC's legitimacy in governing international peace, security, and order.

"N-5, 2nd Floor, Bajrang House, South Extension-1, New Delhi Pin Code: 110049" |+91-7676564400| https://www.toprankers.com Page 10 of 40
Passage (Q.45-Q.51): Pakistan inaugurated a $2.7 billion nuclear reactor, providing some relief as the nation
grapples with an energy crisis. The 1,100 megawatts capacity power plant will generate some of the nation’s
cheapest electricity, according to data from regulator National Electric Power Regulatory Authority. The facility
was connected to the grid last March.
It’s the second unit at the Karachi nuclear power plant to use a Chinese-designed Hualong One reactor. China
financed the facility’s expansion. Pakistan “badly needs” clean and cheap sources of energy, whether it is
nuclear, hydro, or other renewables, Prime Minister Shehbaz Sharif said at a ceremony at the power plant in
Karachi.
While the new facility is a welcome addition to Pakistan’s stretched grid, it won’t do much to curb dependence
on expensive fossil fuel imports or solve the nation’s chronic energy shortages. Pakistan is in discussions with
the International Monetary Fund to increase its energy tariffs in return for bailout funds. The country’s foreign-
exchange reserves fell to the lowest in nine years last month, as high fossil fuel costs put pressure on the
government’s budget.

45. China-Pakistan Economic Corridor was launched in which of the following year?
(a) 2013 (b) 2015 (c) 2017 (d) 2019

46. Which of the following is TRUE about the China-Pakistan Economic Corridor?
(a) China-Pakistan Economic Corridor is a project under Belt and Road Initiative.
(b) It aims to link the Western part of China (Xinjiang province) to the Gwadar Port in Balochistan.
(c) It will pave the way for China to access the Middle East and Africa from Gwadar Port, enabling China
to access the Indian Ocean.
(d) All of the above

47. Which of the following is NOT TRUE about Gwadar Port?


(a) It is a deep seaport located on the Arabian Sea in Gwadar.
(b) Pakistan purchased the Gwadar enclave from Oman.
(c) It officially became part of Pakistan in 1998.
(d) Gwadar Port is situated at the mouth of the Persian Gulf, Strait of Hormuz.

48. What is the length of the China-Pakistan Economic Corridor?


(a) 1000 km (b) 1500 km (c) 3000 km (d) 8000 km

49. How does the CPEC benefit Pakistan?


(a) By creating new job opportunities (b) By improving the energy sector
(c) By increasing foreign investment (d) All of the above

50. What is the significance of Gwadar Port in the CPEC project?


(a) It is a major seaport in Pakistan
(b) It is a hub for the transportation of goods between China and Pakistan
(c) It is a key component of the Belt and Road Initiative
(d) All of the above

51. Which initiative was launched by EU as an alternative to BRI?


(a) Global Gateway (b) Loss and Damage
(c) Global Goal (d) Start Age

"N-5, 2nd Floor, Bajrang House, South Extension-1, New Delhi Pin Code: 110049" |+91-7676564400| https://www.toprankers.com Page 11 of 40
Passage (Q.52-Q.58): To boost the solar power installation in rural areas under Pradhan Mantri Kisan Urja
Suraksha evam Utthaan Mahabhiyan (PM-KUSUM) PM-KUSUM scheme, the government has extended it till
March [1].
The extension will help in compensating for the impact of a pandemic on PM-KUSUM's implementation. The
center launched this scheme in [2] to add solar capacity of 30,800 MW by 2022. The government had planned
to infuse ₹34,422 crore of funding in the scheme.
The Minister of New and Renewable Energy, R K Singh, said the ministry had conducted a third-party evaluation
of the scheme, where it was found that the it could not bear desired result till now due to the COVID uncertainty.
Hence, following the third-party recommendation, the centre will extend the scheme till [1].

52. Which of the following will replace [1] in the above passage?
(a) 2024 (b) 2026 (c) 2028 (d) 2030

53. Which of the following will replace [2] in the above passage?
(a) 2016 (b) 2018 (c) 2019 (d) 2021

54. What is the main objective of PM-KUSUM scheme?


(a) To increase access to electricity in rural areas for agriculture and other rural activities
(b) To create job opportunities in the installation, maintenance and operation of solar power projects
(c) To reduce dependence on grid in grid-connected areas
(d) All of the above

55. How does PM-KUSUM incentivize farmers?


(a) By allowing them to sell surplus solar power to the grid
(b) By providing subsidies for setting up solar power projects
(c) By providing access to financing for solar power projects
(d) By providing access to equipment for solar power projects

56. What is the main challenge associated with the implementation of PM-KUSUM?
(a) Financial and logistics issues (b) Depleting water levels
(c) Regulatory barriers and stability issues (d) All of the above

57. Who among the following is the minister of New and renewable energy?
(a) Dr Jitendra Singh (b) R K Singh
(c) Bhupendra Yadav (d) Ashwini Vaishnav

58. Which of the following statements is/are correct about PM - KUSUM Yojana?
(a) Government will provide maximum subsidy of 60% to farmers
(b) 30% of the cost will be given by Government in form of loans
(c) Farmers will only have to give 10% of the total cost of the project
(d) All of the above

"N-5, 2nd Floor, Bajrang House, South Extension-1, New Delhi Pin Code: 110049" |+91-7676564400| https://www.toprankers.com Page 12 of 40
Passage (Q.59-Q.65): The Paris Club of creditors formally supports debt restructuring for Sri Lanka, bolstering
the bankrupt nation’s efforts to unlock a $2.9 billion bailout from the International Monetary Fund, according to
people familiar with the discussions.

The IMF received the assurance from the informal group — comprising mostly rich, western bilateral creditors
— in recent days, according to the people who asked not to be identified because they didn’t have permission to
speak publicly. One of them said that the Paris Club drafted a communique last week. The IMF didn’t
immediately respond to requests for comment.

The Paris Club’s backing clears another hurdle for Sri Lanka, which aims to get the IMF loan by the end of
March. That would help to mend an economy that has fallen into recession amid sky-high inflation and elevated
borrowing costs, and pave the way for further assistance.

59. How many debt agreements has the Paris Club reached with debtor countries?
(a) 400 agreements (b) 200 agreements (c) 478 agreements (d) 300 agreements

60. What is the total debt treated in the framework of Paris Club agreements?
(a) USD 300 billion (b) USD 400 billion (c) USD 600 billion (d) USD 614 billion

61. Why has the importance of the Paris Club declined in recent years?
(a) Due to the decrease in the number of member countries
(b) Due to the decrease in the number of debtor countries
(c) Due to the emergence of China as the world's largest bilateral lender
(d) Due to the emergence of Russia as the world's largest bilateral lender

62. Who are the largest bilateral creditors of Sri Lanka?


(a) France, Germany, and the United Kingdom
(b) China, India, and Japan
(c) Australia, Canada, and the United States
(d) Belgium, Denmark, and Norway

63. What is the objective of Paris Club in solving payment difficulties faced by debtor countries?
(a) To provide financial assistance to creditors
(b) To help debtors in political fallouts
(c) To find sustainable debt-relief solutions
(d) To promote economic cooperation between members

64. What is India's position on bilateral negotiation with Sri Lanka in January 2023?
(a) It is against the principle of the neighborhood first
(b) It is in favor of leaving Sri Lanka to fend for itself
(c) It has written to the IMF providing the necessary financial assurance
(d) It is indifferent to the situation

65. How many members are there in the Paris Club?


(a) 18 (b) 20 (c) 22 (d) 25

"N-5, 2nd Floor, Bajrang House, South Extension-1, New Delhi Pin Code: 110049" |+91-7676564400| https://www.toprankers.com Page 13 of 40
SECTION - C: LEGAL REASONING

Directions(Q.66-Q.105): Read the comprehension and answer the questions:

Passage (Q.66-Q.70): The meaning of the word ‘Writs’ would be a command in writing in the name of the
Court. Both the Supreme Court and the High Court have been given the power to issue the writs under Article
32 and Article 226 of the Constitution of India. (a) Habeas Corpus: This is a Latin term which means that you
may have the body. This is available when the person has been detained by an authority without giving any
reasons as to the detention. In simple words, it is basically a remedy for a person who has been unlawfully
detained. The custody of the person may be a public custody or private custody. This means that even if a person
has wrongfully detained some other person in his house, then also a petition for Habeas Corpus can lie. This can
be filed by some other person on his behalf. (b) Mandamus: It is an order by the Superior Court commanding
any authority including the Government Corporation to do or not to do a duty. It lies when public authority has
failed to do his duty. This writ does not lie against private individuals. (c) Prohibition and Certiorari: These writs
are filed against the Courts. Here the Superior Courts would be issuing orders to the inferior courts, where the
courts are acting without their jurisdiction or exceeding their jurisdiction. Sometimes, it is also used when the
rules of Natural Justice have not been followed. Both of them are similar in the sense that they are issued against
the inferior courts but the difference between them is that when the matter is still pending and the Court is having
no Jurisdiction, there would be a writ of prohibition but where the decision has been given; the writ of certiorari
would lie. (d) Quo Warranto: By using this writ, an officer or an authority is asked by what mandate or warrant
he is holding that office. It is a writ to prevent a person from holding an office which he is not legally entitled to
and obviously it would be a public office.

66. Which of the following statement is not in line with the author’s argument about writs?
(a) Under Articles 32 and 226 of the Indian Constitution, the Supreme Court and the High Court are each given
the authority to issue writs.
(b) When a person has been detained by a government official without being given a reason, Habeas Corpus
may be used.
(c) If the lower courts and subordinate legislators go outside their scope of authority, petitions for prohibition
and certiorari are filed against them in Superior Courts who then issues orders to them.
(d) The writ of prohibition would be issued when the case was still pending and the court lacked jurisdiction,
but the writ of certiorari would be issued in cases when the decision had already been made.

67. H was denied the right to vote in the state of Rajasthan because he belonged to a tribal community. He tried
everything he could to persuade the officer of his rights, but in vain. As a result, he filed a complaint in the
district where he was denied the right to vote. The court's decision was against him. He then filed a complaint at
the Rajasthan High Court, but the ruling was once again unfavorable to him. As a last resort, he proceeded to the
Supreme Court, where he was granted justice. Because the matter belonged to the state of Rajasthan, the High
Court of Rajasthan was dissatisfied with the Supreme Court's decision and issued the writ of certiorari in
accordance with Article 226 of the Constitution. Decide.
(a) The High Court lacks the authority to petition the Supreme Court via writ of certiorari.
(b) Article 32 requires the High Court to issue a writ of certiorari against the Supreme Court.
(c) If the High Court considers that the Supreme Court has decided a case outside of its jurisdiction, it is correct
to issue a writ of certiorari against the Supreme Court.
(d) Since the requirements for issuing a writ of certiorari against the Supreme Court are met in this case, the
High Court is correct to do so.

"N-5, 2nd Floor, Bajrang House, South Extension-1, New Delhi Pin Code: 110049" |+91-7676564400| https://www.toprankers.com Page 14 of 40
68. W was appointed as a lecturer at Presidency Private College. He had been working there for the past 20 years
and had done a phenomenal job. As a result, he was promoted to the position of college dean. Because the
incumbent dean was dissatisfied with W's qualifications, he went to court and requested that the Quo Warranto
writ be issued. Comment on the legality of the dean's line of action.
(a) Because a writ of quo warranto can be used to prevent someone from holding a position for which they are
not legally qualified, the dean made the correct decision.
(b) The dean acted inappropriately since the writ of quo warranto cannot be issued in this case.
(c) The dean may petition the court for a writ of quo warranto if the circumstances of the case match the
requirements.
(d) The dean's tenure at the institute had come to an end. As a result, he has no authority to question W's
qualifications.

69. P was hosting her daughter's graduation celebration at her home. She had 400 people invited. She requested her
housekeeper K to come early that morning to assist her with all of the preparations. Even at 8 p.m., she refused
to let her housekeeper go home. Considering she had a few visitors staying the night, she asked her housekeeper
to return home the next morning. K's daughter had exams the next day and was expected to return home that
night. Due to her being compelled to remain at home, her daughter petitioned the court for a writ of habeas
corpus. Decide.
(a) The court cannot issue the writ since only the person being held may file the petition.
(b) P had just restrained her in order to seek her assistance in attending the guests, hence the court cannot grant
the writ.
(c) The daughter's claim will be denied since it was expected that K would have to stay till the next morning if
P had a party.
(d) Considering K was wrongfully detained, the courts have the authority to issue a petition of habeas corpus.

70. When Y was accused of demanding dowry from his wife T, he used violence against her and forbade her from
eating for a few days. T's parents filed a complaint against Y at the High Court. Even after all of the evidence
against Y was presented, the court ruled in Y's favor. It was eventually revealed that the judge was Y's uncle,
and hence the decision was prejudiced. As a result, T's parents petitioned the Supreme Court. What are the
Supreme Court's options?
(a) Since the respective High Court lacked jurisdiction to hear the case, the Supreme Court can issue the writ
of prohibition.
(b) The Supreme Court may require T's parents to withdraw their case because such concerns are meant to be
resolved in family court.
(c) Because the respective High Court lacked jurisdiction to hear the matter, the Supreme Court may issue a writ
of certiorari.
(d) Given that the High Court's decision in the case was valid, the Supreme Court lacks discretion.

Passage (Q.71-Q.74): Any person with the intention to take a movable property out of the possession of the
property holder without his consent is said to be committing theft.Ingredients:
(a) Dishonest intention to take property - The intention is an important ingredient to perform the crime of theft
and the intention must be dishonest. The intention must exist that the taker of the property intends to cause
wrongful gain to one person or wrongful loss to another person. It is not necessary that the taker of property must
have wrongful gain. It would be sufficient if it causes wrongful loss to the property holder. (b) The property must
be movable - The property which is permanently attached to the earth is known as immovable property and
which is not attached to earth and can move from one place to another is known as movable property which is
defined under section 22 of IPC. The subject matter of theft must be a movable property. (c) The property should
be taken out of the possession of another person - For any act to come under the crime of theft,the property must
be taken out of the possession of the property holder without his consent. (d) The property should be taken
without the consent of that person - In order to commit theft, the property must be taken without the consent of

"N-5, 2nd Floor, Bajrang House, South Extension-1, New Delhi Pin Code: 110049" |+91-7676564400| https://www.toprankers.com Page 15 of 40
the property holder. (e) The property must be removed from the original possession - The offence of theft is
complete when there is dishonest moving of property.Whoever commits theft shall be punished with
imprisonment of either description for a term which may extend to three years or with a fine or with both.

71. Z went to her aunt's house to watch for her because she had a high fever. She planned to remain there for a few
weeks. In order for her to recover quickly, she was instructed to give her the medications on time. She and her
aunt got into a major argument one day, and out of frustration, she neglected to give her aunt the medication.
Her uncle gave her a severe scolding. She became further incensed as a result, destroyed all of her aunt's
medication kept in her aunt’s room, and left her home to return to her own. Her aunt had to go to the hospital as
a result. Her uncle accused her of stealing after learning about her act. What do you think about this incident?
(a) Since Z had no intention of stealing, she shall not be held accountable.
(b) As there was no unjust gain on Z's side, she shall not be held accountable for theft.
(c) Z will be made responsible for theft because her actions meet the criteria for theft.
(d) Z will be held accountable for the theft because of how she triggered her aunt's illness.

72. T was supposed to board a flight at 10 p.m. He was in a hurry because he had already been late. In his haste, he
grabbed his servant's spectacles because they were so similar. T had only given his servant the spectacles on the
occasion of Diwali, thus their models were identical. This caused a number of issues for the servant since he
couldn't go to work because of his impaired eyesight. As a result, he filed a theft charge against his master T.
(a) T shall be held accountable for theft because he dishonestly took his servant's spectacles.
(b) T will not be held accountable because it was he who gave his servant the spectacles.
(c) Since a master cannot be held accountable for taking anything from a servant, T shall not be held responsible.
(d) T shall not be held liable since the circumstances in this instance do not meet the requirements of theft.

73. W traveled to the other city to visit his grandmother. He realized that he needed a jacket to wear during Shimla's
chilly winter while he was packing his luggage. To borrow one for a few weeks, he therefore contacted his friend
P. That particular brown leather jacket was what he was after. P asked him to take it out of his cupboard since
he wasn't home. W was stopped from taking the jacket when he got to his house by P, who afterwards accused
him of theft. Find out if her accusations are valid.
(a) Given that W entered P's property without permission, the accusations against him are valid, and he should
be held accountable for theft.
(b) Due to the fact that W already had P's consent, the accusations against him are invalid.
(c) The accusations against W are legitimate since he should be held accountable for trespass because he entered
the premises without authorization.
(d) The accusations against W are unfounded because borrowing between friends is never a crime.

74. R had just bought a gown to wear to a friend's wedding celebration. Another friend X, who was not on good
terms with R, admired the same design and intended to take it from R. X decided to remove it from her bag
because they were already at odds. She intended to grab it from R's suitcase at the wedding venue when everyone
else was enjoying the dance. She couldn't do it alone, so she requested F for assistance. R overheard the entire
arrangement while they were preparing it and eventually filed a theft charge against X. Decide.
(a) X shall be held accountable for theft since she had a dishonest purpose to inflict wrongful loss to R.
(b) X shall not be held accountable because the requirements of theft are not met in the particular circumstance.
(c) Instead, X will be charged with robbery because she orchestrated the entire deal with S.
(d) X is not accountable for theft because she only intended to borrow the dress for the wedding.

"N-5, 2nd Floor, Bajrang House, South Extension-1, New Delhi Pin Code: 110049" |+91-7676564400| https://www.toprankers.com Page 16 of 40
Passage (Q.75-Q.79): Section 308 of IPC provides that whoever does any act with such intention or knowledge
and under such circumstances that, if he by that act caused death, he would be guilty of culpable homicide not
amounting to murder; and in case any hurt is caused to any person by such act, then the accused is liable to be
punished with imprisonment of either description for a term which may extend to seven years, or with fine, or
with both.

Section 324 of IPC, on the other hand, criminalizeswillful infliction of injuries on another and states that whoever
voluntarily causes hurt by means of any instrument for shooting, stabbing or cutting, or any instrument which,
used as a weapon of offence, is likely to cause death, would be punished with imprisonment of either description
for a term which may extend to three years, or with fine, or with both.

In contrast to Section 308 IPC, which necessarily requires proving intention or knowledge, to attract Section 324
IPC it is sufficient if a person voluntarily causes hurt by means of an instrument for stabbing or cutting.Under
the former (Section 308), injuries must be such as are likely to cause death, but in the latter (Section 324) the
injuries may or may not endanger one’s life.

Applying the law to the facts of the Roop Chand @ Lala v. State (NCT) of Delhi case, the bench observed that
the evidence on record falls short of establishing the requisite ingredients of Section 308 of IPC, though the
accused is guilty of voluntarily causing hurt with a sharp-edged weapon within the meaning of Section 324 of
IPC. The court also observed that the incident also doesn’t reflect any mental depravity or criminal instincts on
part of the accused and that he appears to be a poor labourer, and has not misused the concession of bail granted
more than ten years back.

75. Ali and Abdul were two brothers. While Abdul was the meritorious one, Ali was notorious and was therefore
always yelled at. Abdul always excelled at everything he did and was recently awarded a scholarship to study
abroad. Jealous Ali thus began fighting with Abdul for being overachiever. In one such fight, Ali penetrated the
tip of a small needle in Abdul’s finger, which got infection due to the rust in the needle. As a consequence of the
infection Abdul died a few days later. Choose the correct option.
(a) Ali shall be charged under section 324 as he has voluntarily caused hurt by stabbing Abdul with the needle.
(b) Ali shall not be charged under section 324 as he has not caused hurt on Abdul with a weapon that is likely
to cause death to Abdul.
(c) Ali shall not be charged under section 324 as he acted in grave and sudden provocation due to his brother’s
over achievements.
(d) Ali shall be charged under section 324 as he had the knowledge the wound could possibly be fatal and cause
death to his brother.

76. Shasha and Shanaya were two ballerinas in the same group who were competing for the position of the lead
ballerina. One fine day, Shasha cut the ribbon of the shoes of Shanaya so that she falls before the audience during
her performance. Due to this mischief, Shanaya falls from the stage during her performance and breaks a bone
of neck. As a consequence to these injuries she dies in the hospital. Decide whether Shasha shall be punished
under section 308 or not?
(a) Shasha shall be punished under section 308 as she voluntarily did the act knowing that if it causes death she
could be charged with culpable homicide not amounting to murder.
(b) Shasha shall be punished under section 308 as she had the intention of causing hurt to Shanaya so that she
could eliminate her competition for the position of lead ballerina.
(c) Shasha shall not be punished under section 308 as she had neither the knowledge and nor the intention to
cause death to Shanaya.
(d) Shasha cannot be held liable as she just cut the ribbons of Shanaya’s shoes and such a meager act cannot be
punished even if it leads to the death of a person.

"N-5, 2nd Floor, Bajrang House, South Extension-1, New Delhi Pin Code: 110049" |+91-7676564400| https://www.toprankers.com Page 17 of 40
77. Samuel is an iron smith known for making the best swords in the kingdom of Fortuna. He is also known in the
adjacent kingdoms for his fencing skills and is therefore requested to teach the kids of the Royal Families. In
one such class he mistakenly inflicted a wound on the arm of the prince of a kingdom. He was arrested for
voluntarily causing hurt to the prince who was likely to cause his death. Now Samuel is charged under section
324 and is being tried. If the above laws apply to the kingdom of Fortuna, decide which of the following is the
correct answer?
(a) Samuel cannot be tried for offence under section 324 as for a person to be charged under section 324 the hurt
shall be caused voluntarily. Samuel did not cause the wound to the prince voluntarily.
(b) Samuel can be tried for offence under section 324 as the weapon used by Samuel is very likely to inflict fatal
wounds or cause death to a person.
(c) Samuel cannot be tried for offence under section 324 as the offence was not committed with any intention
or knowledge of causing hurt or death to the prince.
(d) Samuel can be tried for offence under section 324 as he should have been extra cautious while training. The
act of being negligent is the same as causing hurt voluntarily.

78. Ben was a college student who had affection towards a batch mate Kevin. He proposed Kevin who turned down
the proposals. However, he never stopped stalking Kevin. When he found out that Kevin was dating someone
else, he was enraged. He stopped Kevin on a street at night and upon an altercation, lifted a stick lying around
and hit him in the head with a blunt object. Kevin died on the spot and Ben fled the crime scene. When caught,
Ben confessed that he had no intention of causing grievous hurt or causing death, however, he knew that he
might kill Kevin by hitting him. Whether Ben be charged for culpable homicide not amounting to murder?
(a) Ben cannot be charged under section 308 as he had no intention to kill Kevin and only acted in a fit of rage.
(b) Ben can be charged with section 308 as he had the knowledge that the blow might kill Kevin even if there
was no intention to do so.
(c) Ben cannot be charged under section 324 as the weapon used to inflict wound or cause hurt, was not a sharp
object that could be used to stab or cut.
(d) Ben can be charged under section 324 as Ben voluntarily hurt Kevin by hitting him with a blunt object on
the head.

79. Lakshya was a clerk at an office, and was trying for a promotion for years. He was diligent and efficient in his
work but missed on appraisals and promotions in the office. He very soon realised that he was deliberately being
denied promotions because of biases of his boss. He was distressed because of job and meager salary that could
not make his ends meet. Under the same distress he picked up one of his office knives and stabbed his boss when
both of them were alone in the office. Next morning he surrendered and admitted to stabbing the boss voluntarily.
The boss did not however die. Decide whether Lakhya can be charged under section 324 or not?
(a) Lakshya can be charged under the section 324 of the IPC as wounds were voluntarily inflicted with a weapon
that was likely to cause the death of his boss.
(b) Lakshya cannot be charged under the section 324 of the IPC as he had neither the intention nor the knowledge
that the stabbing is likely to cause harm to or death of his boss.
(c) Lakshya can be charged under the section as a person can be reasonably presumed to know that stabbing can
be fatal and can result in the death of a person.
(d) Lakshya cannot be charged under the section as his act was not voluntary but a result of rage and provocation.
Therefore he cannot be charged under any offence.

"N-5, 2nd Floor, Bajrang House, South Extension-1, New Delhi Pin Code: 110049" |+91-7676564400| https://www.toprankers.com Page 18 of 40
Passage (Q.80-Q.85): The Supreme Court recently reiterated that the highest bidder in an auction does not have
any legal and equitable right to claim the auctioned property unless the same is approved by the State government
or any other authority.
It was argued by the appellant that the possession of the respondent was said to be an illegal possession since the
same was not approved by the State government. The appellant-committee had argued that mere fact that the
plaintiff was the highest bidder would not confer any equitable and legal right to him and it shall be only after
the confirmation of sale and the letter accepting the bid is issued that the respondent could claim any enforceable
right.
A bid is a slightly more complex form of a contractual transaction wherein an offer is made to pay a specified
amount of money for an article being sold at an auction. It can be termed as an instant contract as the offer and
acceptance take place almost instantaneously or within a span of a few minutes. However, the same rules of
revocation apply i.e. the revocation of the offer must be made before the acceptance of it or before the bid is
knocked down by the auctioneer. Similarly, wherein the bid is only provisionally accepted by the knocking down
of the hammer and is subjected to confirmation by a higher authority the offeror or in this case the bidder has
the option to withdraw the bid before such a confirmation is provided.

There also arises discussion where the bidder makes a default in the payment of the deposit and whether it must
be treated as a revocation of the bid.
The Bench then relied on its recent decision in State of Punjab and Others v. Mehar Din (2022), wherein it had
observed that the government authority is not bound to accept the highest bid in an auction and the acceptance
of highest bid is always subject to conditions of holding public auction.

80. The government of Madhya Pradesh decided to put up for auction a famous Turin shroud which was recently
imported from abroad. As per the terms of the auction, it was mentioned:
 The auction shall be deemed to be concluded and confirmed after the gavel is hit three times by the
auctioneer.
 No bidder is permitted to withdraw his bid post the hitting of the gavel and the contract shall be deemed to
be concluded as binding.
 The signing of the written contract shall only be for the purpose of formality.
 The bid awardee would be required to make the payment after the execution of the contract within one month
of the contract signing, on the failure of which the government would have the right to revoke the offer. The
item can then either be sold to the next highest bidder or the auction can be conducted afresh subject to the
discretion of the Deputy General of the Department, on whose confirmation and intimation, the contract shall
be deemed to be executed.
During the bid, many art lovers joined the queue in the hope of getting the historical masterpiece. However, only
one could be the owner of this shroud. After a long price war, Mr. Mukesh gave the highest bid and the auction
concluded by the hitting of the gavel trice. But before the signing of the bid contract, the government
representative came and informed that due to the factor that some of the major industrialists/ invitees couldn’t
attend the auction, this auction shall be called off and will be re-conducted to realize the value of the shroud. Is
the government correct and is it within its power to re-conduct the auction?
(a) Yes, since no official contract has been entered into between the parties, the government is free to revoke
the bid at any time.
(b) No, the bid was already concluded after the hitting of the gavel and cannot be withdrawn as per the terms of
the auction to which the auction was subjected to.
(c) Yes, the highest bidder in an auction does not have any legal and equitable right to claim the auctioned
property.
(d) No, the purpose of any bid is to realize the highest value for any article/ commodity and thus the government
can re-conduct the auction.

"N-5, 2nd Floor, Bajrang House, South Extension-1, New Delhi Pin Code: 110049" |+91-7676564400| https://www.toprankers.com Page 19 of 40
81. Continuing with the above facts it was the contention of the government that the highest bidder in an auction
does not have any legal and equitable right to claim the auctioned property and hence Mr. Mukesh is not entitled
to any claims over the auctioned property as the contract has also not been signed and executed between the
parties. Is the contention of the government correct?
(a) No, the terms of the auction clearly provided that execution of the contract would merely be a formality and
the contract would be concluded soon after the gavel is hit thrice.
(b) Yes, the highest bidder in an auction does not have any legal and equitable right to claim the auctioned
property
(c) No, in any bid the contract gets concluded in the hitting of the gavel thrice.
(d) Yes, since the official binding agreement has not been entered into by the parties, the contract cannot be
deemed to be concluded and the government has no obligations to honour the bid.

82. In the same facts if After 6 months of continuous litigation, the government realizing that not auctioning the
property would cause more harm monetarily, agreed to end the litigation and sell the Turin shroud to Mr. Mukesh
honouring the previous terms and conditions of the auction. Mr. Mukesh executed the contract with the
government on papers. Mr. Mukesh made half the payments at the time of signing and agreed to pay the other
half within this month. However, Mr. Mukesh suffered severe business losses and couldn’t pay the whole
amount. Unfortunately, despite several requests by Mr. Mukesh, the government decided to revoke the offer. Is
the government entitled to do so?
(a) No, default in payment cannot be the sole ground to revoke the bid and thus the government is incorrect in
taking such actions.
(b) Yes, since the payment is the prime consideration is such an auction, any delay will be detrimental to the
interests of the government (which resumed the sale to Mr. Mukesh to reduce the losses) and hence the
revocation was valid.
(c) No, since Mr. Mukesh had already paid half the amount and it was only due to uncontrollable financial
circumstances he was retrained from making the full payments.
(d) Yes, the terms of the auction clearly illustrate that on the failure of the highest bidder to pay the requisite
amount within one-month time frame, the government can revoke the offer.

83. After the contract was revoked, the next highest bidder, Mr. Anil went all the way to get the item. However, the
Deputy General of the Department decided to conduct the auction afresh due to considerable difference between
the price quoted by the highest bidder and subsequent bidder. Mr. Anil was also ready to pay additional sum
equivalent to such difference, however, the government decided to reconvene the auction. Is the government
correct in re-organizing the auction?
(a) Yes, the Deputy General has the sole discretion to either conduct the auction afresh or to conclude the sale
with the second highest bidder.
(b) No, since Mr. Anil was ready and willing to make up for the difference between the pricing quoted by him
and the highest bidder, his offer should ideally be accepted.
(c) Yes, the organizer of the bid has the sole discretion on whom to offer the bid and whom it should not offer
the bid and such right is unconditional.
(d) No, the Deputy General has to provide cogent reasons before taking any decision and it cannot without
providing such reasons take any action.

"N-5, 2nd Floor, Bajrang House, South Extension-1, New Delhi Pin Code: 110049" |+91-7676564400| https://www.toprankers.com Page 20 of 40
84. It was the contention of Mr. Anil that he was informed by the staff of the Deputy General himself that the Deputy
General has decided on going in favour of the second highest bidder and therefore the contract shall be deemed
to be executed. Is the contention of Mr. Anil correct?
(a) Yes, since the confirmation of the bid came from the sources inside the office, the condition of confirmation
and intimation remains satisfied and therefore the bid is now biding on the parties.
(b) No, since there were no official confirmation from the office of the Deputy General, the contract cannot be
taken to have been formed.
(c) Yes, the Deputy General cannot approbate and reprobate at the same time and once confirmed cannot
withdraw the offer later.
(d) No, the discretion of whether to give the bid to the second highest bidder or to conduct the bid afresh, lies
solely with the Deputy General.

85. Assuming, that Deputy General in the evening of 28th January, 2022 decides to provide confirmation to the
second highest bidder but on 20th January, 2022 Mr. Anil decides to withdraw the bid due to financial reasons.
The communication of revocation reaches the Deputy General’s office in the morning of 28th January, 2022. Can
Mr. Anil withdraw the bid?
(a) Yes, since both the bid revocation and bid acceptance happened on the same day, the revocation of the bid
shall prevail.
(b) No, since the bid was only conditional upon the confirmation by Deputy General and provides for no
provision for revocation, the bid cannot be revoked.
(c) Yes, the bidder has the option to withdraw the bid till the time no confirmation is given the Deputy General
confirming his bid.
(d) No, since both the bid revocation and bid acceptance happened on the same day, the acceptance of the bid
shall prevail.

Passage (Q.86-Q.90): In order for a contract to be valid, the parties must have freely and voluntarily entered
into the agreement with a clear understanding of its terms and consequences. Section 13 of the Indian Contract
Act elaborates that consent is said to be given when both parties to the contract have agreed to the same thing in
the same sense. The same is an implementation of the Latin phrase consensus ad idem, which translates to
‘meeting of minds’. If there is a mistake or misunderstanding regarding a material term of the contract, or if one
party has been induced to enter into the contract by fraud, undue influence, or coercion, then there is no valid
consent and the contract will be voidable. Section 14 of the Indian Contract Act states that "consent is said to be
free when it is not caused by coercion, undue influence, fraud, misrepresentation, or mistake."

Section 15 of the Indian Contract Act defines ‘coercion’ as committing, or threatening to commit, any act
forbidden by the Indian Penal Code, or the unlawful detaining, or threatening to detain, any property, to the
prejudice of any person whatever, with the intention of causing any person to enter into an agreement.
Though, it shall be understood that through judicial precedents the scope of coercion has now been expanded to
other actions that are not particularly offences in the Indian Penal Code, these may include other acts which
coerce the individual to consent to the act. A contract formed by consent given due to coercion is voidable on
the option of the party whose consent was not free.

In situations where consent is obtained through coercion the burden of proof is on the plaintiff claiming that they
were coerced to consent to the contract. A mere suspicion or probability is not enough to support a plea that the
contract was formed under coercion, the same has to be backed by oral or documentary evidence.

"N-5, 2nd Floor, Bajrang House, South Extension-1, New Delhi Pin Code: 110049" |+91-7676564400| https://www.toprankers.com Page 21 of 40
86. A is an avid art collector and possesses numerous paintings from prominent painters all over the world. B is a
contemporary of A and he too is an art collector. B has always been jealous of A’s art collection, and has always
had the intention of buying the same. One day B got to know that A is going through some financial troubles
and is now looking to sell some of the artworks he owns. B met with A in order to purchase the artworks in A’s
gallery. B offered him a lower price than what A was asking for, but in order to convince A, B said “I am the
only buyer you will have after this meeting” and then put a lighter on the table. Decide whether B coerced A to
consent or not:
(a) B coerced A to consent by saying, “I am the only buyer you will have after this meeting”.
(b) B coerced A to consent by putting the lighter on the table, indicating that there may be consequences.
(c) B has not coerced A as he was merely trying to bargain a better price for the deal.
(d) B has not coerced A as neither his statement not his actions are sufficient to establish any sort of coercion in
the situation.

87. Continuing with the similar facts as stated above, suppose A did not consent to B on the terms which were
proposed by him. Aggravated by the same B, took the lighter, lit it and threatened A, that if he did not concede
to B’s conditions of the contract he would burn the whole gallery and then his collection will not have any value.
A reluctantly decided to agree to B’s terms of the contract. Decide whether A’s consent is valid or not:
(a) A’s consent is not valid as the same was given under coercion.
(b) A’s consent is valid as he knew B would not have destroyed such a valuable art collection.
(c) A’s consent is not valid as B forced the consent out of A.
(d) A’s consent is valid as he had no other buyer than B.

88. X was married to Y for 25 years and they had a son named Z who was 20 years old. X started having an extra-
marital affair with P. X had transferred half of his property in the name of Y and their son Z. Though as soon as
his affair started with P, he slowly started rethinking his earlier decision of transferring the property to Y and Z.
X asked Y and Z to return the property that he had earlier transferred, which was refused by Y and Z thinking
that if they do so X will abandon them and leave them on the streets. X in order to convince them X threatened
that he will commit suicide. Fearing the same Y and Z transfer the property back to X. Whether the consent
given by Y and Z is valid:
(a) The consent given by Y and Z is valid as attempt to suicide is not an offence and thus will not invite
application of Section 15 of the Indian Contract Act.
(b) The consent given by Y and Z is not valid as the threat to commit suicide is enough to establish coercion
even if the same is not an offence anymore.
(c) The consent given by Y and Z is not valid as attempt to suicide is an offence within the Indian Penal Code.
(d) The consent given by Y an Z is valid as they gave the same for saving X’s life.

89. P and S were neighbours in a society in Gujarat. S knew that P was a habitual drinker and used to source alcohol
in Gujarat through bootleggers knowing that possession and consumption of the same without government
license is prohibited in the state. P had a beautiful red sportscar that S always fancied. S went to P and told if she
did not sell the sportscar to S at a minimal price S would report to the police that P buys alcohol from bootleggers.
Fearing the complaint P sold the car to S. Whether the consent given by P valid.
(a) The consent given by P is not valid, as the same was obtained through threatening criminal complaint.
(b) The consent given by P is not valid, as she was threatened about S revealing her personal habits to the police.
(c) The consent given by P is valid, as threatening criminal prosecution is not an offence under IPC.
(d) The consent given by P is valid, as P knew she was wrong and just wanted to evade trouble.

"N-5, 2nd Floor, Bajrang House, South Extension-1, New Delhi Pin Code: 110049" |+91-7676564400| https://www.toprankers.com Page 22 of 40
90. G was a businessman in dire need of money, so he pledged one of his antique vases to his friend H for rupees
one lakh without interest. After a while when G was able to return the money to H but H refused to return the
vase unless he pay an extra Rs. 10,000 as interest. Since the vase was his family heirloom G paid the extra
amount to get the same back. G approaches the court demanding recovery of the interest paid as the same was
paid due to coercion. Decide:
(a) The act of H would fall within the ambit of detention of property, thus, the consent obtained was due to
coercion, and is voidable at the option of G.
(b) The act of H is valid as he is entitled to take interest for the amount he had given to G.
(c) The act of H is valid as when the vase was given, he obtained absolute authority over it and could decide as
what will the same be valued.
(d) The act of H is not detention of property, but H cannot change the terms of the contract at the last minute.

Passage (Q.91-Q.95): The Uttarakhand Competitive Examination (Measures For Control and Prevention of
Unfair Means in Recruitment) Ordinance, 2023, has provisions of fines up to Rs 10 crore and life imprisonment
for the guilty. It was urgently approved by CM Dhami and forwarded to Raj Bhavan. With Governor Lt Gen
(retd) Gurmit Singh’s assent, it became law within 24 hours.

The ordinance said that the main aim behind the law was to prevent offences related to obstructing the sanctity
of examinations, use of unfair means, leakage of question papers, and other irregularities. It covers public
examinations for recruitment to posts under the state government, autonomous bodies run by the government,
and authorities, corporations, and institutions operated with grants of the state government.

According to the ordinance, if any examinee is caught cheating or causing another examinee to cheat in a
competitive examination (online and offline) or to have indulged in unfair means, he shall be punishable with
imprisonment for three years and with a minimum fine of Rs 5 lakh. If the fine is not paid, the examinee shall
be jailed for another nine months.
A second-time offender will be punishable with a minimum jail term of 10 years and fine of Rs 10 lakh. In
default of payment of fine, he will be jailed for another 30 months.

91. John was a student who appeared for a competitive examination for admission to a prestigious private university.
During the examination, he was caught cheating by one of the invigilators and was subsequently arrested and
jailed for nine months under the ordinance. What argument according to you will be used by the defence lawyer
here to support his case?
(a) There was no concrete evidence to prove that John had actually cheated during the examination.
(b) They highlighted the lack of any concrete evidence such as confiscated materials, eyewitnesses, or CCTV
footage that could prove that John had actually cheated.
(c) John's lawyer argued that the ordinance under which John was imprisoned was not applicable in this case.
(d) John's lawyer argues that the ordinance under which John was imprisoned was vague and did not provide for
due process.

92. Tom was a student who appeared for a competitive examination for admission to a prestigious private university.
During the examination, he used unfair means to cheat. However, he was caught by one of the invigilators and
his exam paper was confiscated as per the new law. During the trial, the prosecution presents evidence that Tom
used a concealed Bluetooth earpiece to communicate with an accomplice outside the examination center. The
prosecution also presents testimony from the invigilator who caught Tom in the act. Tom's lawyer argues that he
did not use any unfair means during the examination and that he was falsely accused. However, the evidence
presented by the prosecution clearly shows that Tom had used unfair means to cheat during the examination.
Choose a correct statement:
(a) Tom will be held accountable under the ordinance, which punishes any examinee caught cheating in an
examination by unfair means.
(b) Tom will not be held accountable because the ordinance laws do not apply to him.
(c) Tom will be held accountable since the prosecution's evidence plainly reveals that Tom utilised unfair
techniques to cheat during the test.
(d) Tom will be held accountable as the examiner is discovered to have used unfair tactics to cheat in the exam.
"N-5, 2nd Floor, Bajrang House, South Extension-1, New Delhi Pin Code: 110049" |+91-7676564400| https://www.toprankers.com Page 23 of 40
93. Emma appeared for an online competitive exam for admission to Delhi University run by Delhi government,
which was conducted in two phases due to the COVID pandemic. Although she had the convenience of taking
the national entrance exam from her own home, she was aware that the examiner was using an online proctoring
system to monitor any suspicious behavior by students. Emma successfully cleared the exam and secured the top
rank. However, Rachel became jealous of Emma's success and accused her of cheating in the exam by placing
notes on her screen. The allegation would be considered valid only if Rachel can produce evidence to support it.
The issue to be determined is whether Emma is guilty of cheating under the provisions of The Uttarakhand
Competitive Examination (Measures for Control and Prevention of Unfair Means in Recruitment) Ordinance,
2023.
(a) Emma will be held accountable under the stated ordinance, that punishes anyone caught cheating in a
competitive test.
(b) Emma will not be held responsible. Rachel merely claims Emma cheated and has no solid evidence to support
her claim.
(c) Emma will not be held accountable because she was not caught cheating in a competitive examination.
(d) Emma will be held accountable for using unfair means to cheat in a public competitive test.

94. Continuing with a similar scenario, determine in response to the preceding question regarding Emma's
culpability, that this time Emma was taking a state recruitment services examination administered by the
Uttrakhand government and she was detected with some Chits in her stationery box. Notwithstanding the fact
that the Chits collected were unrelated to the topics covered in the examination. Will she be held in accordance
with the terms of the Uttarakhand Competitive Examination (Control and Prohibition of Unfair Methods in
Recruitment) Ordinance, 2023?
(a) She will be held liable as the ordinance punishes an offender who is caught cheating in an examination by
unfair means.
(b) She will not be held liable as the Chits found were not unrelated to the topics covered in the examination.
(c) She will be held liable irrespective of the facts that the Chits found were not related to the topics covered in
the examination.
(d) She will not be held accountable since the circumstances provided do not satisfy Emma's prosecution under
the aforementioned ordinance.

95. What will be Emma’s punishment, as per the culpability of Emma in the previous cases discussed above in above
question:
(a) Since Emma is a second time offender she will be punishable with a minimum jail term of 10 years and fine
of Rs 10 lakh.
(b) Since Emma is a first time offender she shall be punishable with imprisonment for three years and with a
minimum fine of Rs 5 lakh.
(c) It cannot be decide as Emma was not found culpable in either of the two circumstances given above.
(d) Emma will be awarded Punishment under the head of a second-time offender.

Passage (Q.96-Q.100): According to Section 4(1) of the Sale of Goods Act, a contract of sale of goods takes
place when two parties enter into an agreement wherein one party transfers or agrees to transfer a property to the
other party in return of a price that may be paid or promised to be paid, or for some other valuable consideration.
A contract of sale may be absolute, or conditional. If a property is transferred from one party to another, it may
be called a sale but when the transfer of the said property is decided to take place on a future date, it must be
termed as an agreement to sell. If conditions to an agreement to sell are fulfilled in the prescribed time duration
and the transfer of property is complete, it becomes a sale. The completion of sale takes place when the property
ceases to be in the possession of the seller and moves to the buyer and the buyer has accepted it. The property
passes to the buyer if he does not signify his approval or acceptance to the seller but retains the goods without
giving notice of rejection, then, if a time has been fixed for the return of the goods, on the expiration of such
time, and, if no time has been fixed, on the expiration of a reasonable timeIn sale, the ownership rights of the

"N-5, 2nd Floor, Bajrang House, South Extension-1, New Delhi Pin Code: 110049" |+91-7676564400| https://www.toprankers.com Page 24 of 40
goods are passed to the buyer with immediate effect unlike in an agreement to sell where some specified
conditions must be fulfilled. The risk passes with the ownership of the property, not possession. Section 5 of the
Act defines how a contract of sale is made. When a party offers to sell or buy goods in return of a price and the
party to which the offer is made, accepts the offer, an agreement to sale is concluded. The contract for sale may
be oral, or in writing or implied by the conduct of the contracting parties.

96. Raghu and Veer enters a contract where Veer agrees to sell 2kgs of wheat for Rs. 1000 but only after measuring
it. Raghu pays him the amount of wheat and asks him to deliver the wheat to him after doing the measurements.
In the given situation, which of the following statements is true?
(a) The property has been passed to Raghu since he has paid for the wheat.
(b) Since the property has not been passed to Raghu,the sale will be completed only after signing of agreement.
(c) This is an agreement to sale since they have already entered into a contract.
(d) This is an agreement to sell since the delivery is to be made later on.

97. Dhanush visits Rajesh’s shop to buy some sarees for his wife Nidhi. After entering his shop, he gets confused
by looking at a huge range of sarees and so asks the shopkeeper to help him. He then selects three sarees and
asks the shopkeeper to allow him to take the sarees home so that he can get his wife’s suggestion and then finalise
a saree and buy it. Rajesh agrees to this. At home, Dhanush’s son drops a glass of milk on one of the sarees. Who
will bear the risk in this situation? Decide.
(a) The risk will be borne by Dhanush since the sarees were in his possession.
(b) The risk will be borne by Rajesh since the sarees were his property.
(c) The risk will be borne by Dhanush since they got ruined because of his son.
(d) The risk will not be borne by anyone since the damage happened only due to mistake.

98. Raghav sends a mail to Rashi expressing his wish to sell his Maruti Suzuki to her for Rs.1 lakh. Rashi replies to
the mail saying that she is interested but would first like to test the car for 3 days. Raghav agrees to the same and
tells her that he will send his car to her and will wait for 3 days after which he will assume that she has accepted
the sale. Raghav sends the car to her on 20th March but Rashi does not respond to even him even till 25th. Has
the sale been completed in this scenario? Decide.
(a) Yes, since Rashi did not respond after 3 days.
(b) No, since she did not actually tell him her acceptance to buy the car.
(c) Yes, since mere sending car for trial amounts to sale.
(d) No, since mere silence does not mean acceptance.

99. Arina visits Radhika’s shop to buy a pack of Biscuits. Arina asks Radhika about a specific type of biscuits to
which Radhika says that she currently does not have them in shop but have to get them from their godown and
can only give her after 2 hours but asks her to pay for the same. Arina pays for the biscuit and asks her to deliver
them as soon as they are available. In the given situation, which of the following statements is true?
(a) This is an agreement to sell since the delivery was to be made later on .
(b) This is an agreement to sell since there was a condition present.
(c) This is a contract of sale since the property has been passed to Arina.
(d) This is a contract to sale since Arina had already paid the amount.

100. Amit, a fitness freaks only consumes egg laid by a special imported Pakistani Hen. He visits Qureshi’s poultry
shop and asks him for a dozen eggs. Qureshi informs him about the bad health of the hen and says that he will
deliver him the eggs as soon as the hen lays them. Amir agrees and pays him half the amount and goes back
home. In the given situation, which of the following statements is true?
(a) This is an agreement to sell since there is a future transfer of property.
(b) This is an agreement to sell since there is a future delivery of goods.
(c) This is a contract of sale since Amit had already paid half of the amount.
(d) This is a contract of sale since Amit had placed an order already.

"N-5, 2nd Floor, Bajrang House, South Extension-1, New Delhi Pin Code: 110049" |+91-7676564400| https://www.toprankers.com Page 25 of 40
Passage (Q.101-Q.105): Negligence is a type of tort which means a breach of duty (duty to take care) by one
person which causes damages to another person. It is an act of carelessness and ignorance on the part of the
defendant, which he is obligated to perform, which a rational and prudent man would not do. In general,
negligence is the omission to perform a duty which results in the plaintiff’s injury. Negligence is committed in
respect of both person and property. Breach of duty to take care and measures in order to avoid any kind of
performing an act is the basic requirement in order to raise liability of negligence. Suit for negligence arises
when there is a breach of duty which is recognised by law. The term duty of care in itself says duty or obligation
to be more careful, reasonable and rational so that it may not give rise to any kind of injury or damage. Generally,
if there is any kind of breach in the performance of such duty it gives rise to the liability of negligence for which
compensation is to be paid. The degree of care depends upon the gravity of injury an act possesses. If an act in
which any kind of omission or ignorance is likely to cause injury to a greater extent more care is required and if
the danger is slight less care is required. In order to determine whether a person is liable for negligence is loosely
based on his rationality to act on that particular situation. Essentials of negligence are: The defendant shall be
legally obligated to perform such duty, Ignorance of the defendant in order to take reasonable care while
performing his act, Injuries must be sustained by the plaintiff due to the carelessness of the defendant. Plaintiff
has the burden of proof. He must prove that he has sustained injury from the negligence of the defendant and
their stands a proximate relation between them.

101. M was already 15 minutes late to his office. He chose a shorter route in order to get to the office faster. However,
as he was exiting one lane, he noticed that one side of the road was blocked due to construction, so he started
driving on the opposite side of the road. He made a deliberate effort to cross that lane without risking himself or
anyone else, but as he reached the other end, he came across a biker who was traveling in the opposite direction
of M and made an abrupt turn. He made a turn to avoid an accident, but unfortunately his car hit a road divider
and he sustained injuries. Will M be accountable for his carelessness? Choose the best response in the light of
a given passage.
(a) There was no legal duty of M to begin in the first place to take care of the biker, hence the liability of
negligence cannot be attracted.
(b) M will not be held liable under negligence because the biker took a sudden turn to lane and hence it should
be him who should be held liable.
(c) M did not take reasonable care and was involved in a kind of omission or ignorance which was likely to
cause injury to any third person.
(d) M will not be held liable under negligence because he did not cause any injury or harm to the plaintiff.

102. The government gave a contract to a road-building business to build 500 kilometres of road. They owed it to the
public to carefully construct the road so as to prevent any injuries. To complete the job earlier, they used road
tar without first checking its quality during construction. J, a truck driver, was using that road one day after it
had been built entirely with big loads on his vehicle, even going beyond the speed limit. Unfortunately, a car and
his large truck collided, resulting in a serious disaster. A negligence claim was filed against the construction
company. Choose the best response in the light of the given passage.
(a) The Road Company will not be held liable because it was government who has duty to supply proper quality
of services to its public.
(b) The Road Company has no legal obligation towards the public hence it will not be held liable for the accident
that took place between truck and car.
(c) Road Company will be held liable because it was negligent in choosing the road tar and used one that was
not of good quality
(d) Truck driver will only be held liable because he carried such heavy items on his truck which led to collision
with the car.

"N-5, 2nd Floor, Bajrang House, South Extension-1, New Delhi Pin Code: 110049" |+91-7676564400| https://www.toprankers.com Page 26 of 40
103. N was returning from a club after partying, and he was drunk. There was a "no smoking" sign at gas station,
which he did not see, and he lit a cigarette at gas station, but there was a petrol leak, and it caught fire with the
spark of the cigarette. The gas station sued N for carelessness, but he responded by asserting that the gas station
had broken its legal obligations. . Choose the best response in light of the given passage.
(a) N will not be held liable since he was not legally obliged to fulfill duty of care as he was unaware of it.
(b) Petrol pump will be held liable because due to its negligence, a petrol leak was there and that led to fire there.
(c) N will be held liable because he did not take reasonable care to avoid the mishappening.
(d) Petrol pump will not be held because under negligence it is essential that a person must be legally obliged to
perform its duty.

104. P was driving a car towards his village, and after smoking he threw the cigarette bud in the road, which caused
a fire in a nearby field of a farmer. Not only did the crops in that field burn in the fire, but L, a farmer who was
working the field's corners, also sustained some body burns. L's son F filed a negligence claim against P. In light
of the passage, select the best answer.. Choose the best response in the light of the passage.
(a) P’s irresponsible behavior did not lead to grievous burn to a person but to property hence he will not be held
liable.
(b) F will be able to prove that it was P who committed negligent act as proof of burden will be on him hence P
will be held liable.
(c) P will be held liable since under negligence it is necessary that a person negligently ignored his duty of care
and also injured a person.
(d) P will not be held liable because under negligence the plaintiff must sustain injuries due to negligent behavior
of the defendant.

105. Which of the following statement is not in line with the author’s contention as given in the passage?
(a) The fundamental prerequisite for establishing negligence responsibility is a failure to exercise reasonable
care.
(b) The phrase "duty of care" itself denotes a responsibility or obligation to exercise greater caution, reason, and
common sense in order to prevent any harm or damage from occurring.
(c) The defendant will be required by law to fulfil his duty with care and if the defendant fails to exercise
reasonable care while doing his conduct, he will be held accountable.
(d) The onus of proof is on the defendant and he must demonstrate that the injury to him was caused by the
plaintiff's negligence and that there is a direct link between them.

"N-5, 2nd Floor, Bajrang House, South Extension-1, New Delhi Pin Code: 110049" |+91-7676564400| https://www.toprankers.com Page 27 of 40
SECTION - D: LOGICAL REASONING

Directions (Q.106-Q.135): Read the passage given below and answer the questions that follow-

Passage (Q.106-Q.111): The Government’s summit for developing nations, called the “Voice of the Global
South Summit”, as its first big leadership-level G20 event, is an extremely important signal. It is also a departure
from New Delhi’s looking towards the “high-table” of global leadership, involving its relationship with the
UNSC P5 and G-7 (the most developed economies), to focusing on a more just view of the world and how the
developing world is being affected by global inequities. In his opening remarks at the virtual summit, Prime
Minister Narendra Modi explained the reasons for the shift: how “challenges of the COVID pandemic, rising
prices of fuel, fertilizer and foodgrains, and increasing geopolitical tensions have impacted our development
efforts”. External Affairs Minister S. Jaishankar, too, spoke of India’s need to envision a common future with
the Global South and acknowledge India’s “common past” with the Global South, many of whom have suffered
colonialism. Over 10 different sessions, India and representatives of 125 countries, of the 134 that make up the
G-77, agreed that the key issues include the fragmentation of the international landscape, shortages in grain
exports, oil and gas, and fertilizer as a result of the Ukraine war, and terrorism. Of note was Mr. Modi’s push for
“human centered” globalization countering the “first world’s” view of expediting climate change goals at the
cost of development, ensuring immigration and work mobility for skilled populations of the global south, and
resilient renewable energy access. The summit appears to mark a reset in India’s foreign policy outlook in its
year as G20 president: one which has made the Government reclaim the true meaning of non-alignment, in the
wake of the Ukraine war where it refused to take sides. Minister of State for External Affairs Meenakshi Lekhi’s
visit to Cuba as it took over G77 chairpersonship (a grouping India has shunned) and the invitation to the
President of Egypt, a NAM co-founder, as Republic Day chief guest were significant too.

Of note were some of the summit’s exclusions: Pakistan and Afghanistan. Of note too was the inclusion of
Myanmar, whose junta regime has not been recognised but with which India has chosen to forge closer ties. It is
hoped that the collective South-South understanding of global issues will lead to a more inclusive meeting at the
summit level, particularly with reference to South Asian and the subcontinent’s regional problems. It is also of
note that the grouping did not release a common or joint statement, and much of the narrative on outcomes is
built on what Mr. Modi and Mr. Jaishankar said. For India to be heard as the ‘Voice of the Global South at the
G20’, it must reflect on the aspirations of the other nations and amplify them, as a true leader of the developing
world at the G20 summit later this year.

106. What is the best representation of the main idea of the passage?
(a) As the global leader, India would do well to include as many countries as possible and present their views at
the world stage.
(b) India is going too ahead in the race to lead the global south; it must address its internal issues first.
(c) As the leader of the global south, India’s responsibility is to amplify the views of the South Asian countries.
(d) It is important for India to stage its leadership in a particular part of the world before positioning it as a world
leader.

107. At the "Voice of the Global South Summit," the Prime Minister of India, Narendra Modi, emphasized the
importance of a "human-centered" globalization in order to counter the negative impact of climate change on the
developing world. Based on this information, which of the following is the most likely inference?
(a) The Prime Minister of India is against any efforts to address climate change.
(b) The Prime Minister of India believes that developing countries should prioritize economic growth over
addressing climate change.
(c) The Prime Minister of India believes that the developed countries are not considering the impact of climate
change on the developing world.
(d) The Prime Minister of India believes that the developed countries are prioritizing addressing climate change
over the economic development of the developing world.

"N-5, 2nd Floor, Bajrang House, South Extension-1, New Delhi Pin Code: 110049" |+91-7676564400| https://www.toprankers.com Page 28 of 40
108. As per the passage, what could be inferred as qualities of a leader in a geopolitical setting?
(a) A leader must be aggressive and dominating in order to be successful in a geopolitical setting.
(b) A leader must be willing to prioritize their own interests over those of other nations in a geopolitical setting.
(c) A leader must be able to balance their own interests with those of other nations and consider the impact of
their actions on the global community.
(d) A leader must be able to control the actions of other nations in a geopolitical setting.

109. Which of the following is the author of the passage most likely to agree with?
(a) The COVID pandemic has had no significant impact on the developing world and globalization should
continue to prioritize economic growth above all else.
(b) The COVID pandemic has been a blessing in disguise for the developing world and has not affected their
economies.
(c) The COVID pandemic has had a severe impact on the developing world and a “human-centered”
globalization is necessary in order to address the challenges faced by these countries.
(d) The COVID pandemic has had a severe impact on the developing world but it is not necessary to shift
towards a "human-centered" globalization.

110. Which of the following, if true, would most weaken the author's arguments?
(a) The developing world coming together to pull off a solution or agreement to address the challenges arose
after pandemic.
(b) India’s intent provides a collective solution to the problem of South countries.
(c) India's foreign policy is inclusive of strategic interests and the aspirations of other nations.
(d) The developing world has not been affected by the COVID pandemic at all.

111. Given the passage, which of the following statements would play the role of providing additional information to
support the main argument of the passage?
(a) The ramifications of the COVID-19 pandemic on the economies of the developing world are negligible and
inconsequential.
(b) India ought not to participate in the G20 summit, as it is not in the country's best interest.
(c) The developing world already effectively addressed the challenges posed by the COVID pandemic.
(d) India has a long history of inclusive and collaborative philosophy and has always included the interests of
the global south.

Passage(Q.112-Q.117): Why is the Government seeking additional compensation from Union Carbide for the
1984 Bhopal gas tragedy? Way back in 1989, there was an agreement which stipulated a payment of $470 million
(Rs 715 crore at that time) by the American company. That was supposed to be the final settlement. If over three
decades after reaching a settlement, India makes additional demands, what sanctity would be left in the pacts we
make with other countries, big companies, or multilateral agencies? What impact would it have on the foreign
direct investment that our leaders arduously seek? In fact, there are the questions someone in the Government
must have raised when a decision was made to seek an amendment to the mutually agreed 1989 deal with Union
Carbide; apparently, no one did. But the apex court asks questions and flags issues (this is the main reason why
the Government is at loggerheads with the judiciary, but that’s another story). So, it was left to the five-judge
Bench of Justices Sanjay Kishan Kaul, Sanjiv Khanna, Abhay S Oka, Vikram Nath, and JK Maheshwari to
remind the Government about the far-reaching ramifications of the proposed move, one of which would be the
impact on prospective investments by multinational corporations. The Bench is hearing a curative petition filed
by the Centre. Saying that populism cannot be the basis of judicial review and the “sanctity of the settlement”
would go away if the case is reopened, it also said that a Rs 50-crore sum of the compensation which remains
unutilised even after so many years.

"N-5, 2nd Floor, Bajrang House, South Extension-1, New Delhi Pin Code: 110049" |+91-7676564400| https://www.toprankers.com Page 29 of 40
The Government’s move brings to mind the unedifying episode of retrospective taxation which brought a lot of
embarrassment to the nation. The sordid saga of retrospective taxation began with the famous Vodafone case.
Vodafone bought Hutchison’s mobile telephony business and other assets in India in 2007, on which the Indian
tax officials demanded Rs 7,990 crore in capital gains and withholding tax from Vodafone. Vodafone legally
challenged the demand notice, and won the case in the Supreme Court in 2012. The matter should have ended
there, but taxmen managed to convince the then finance minister, Pranab Mukherjee, to disregard the highest
court of the land and undo its ruling by bringing in an ordinance — which he did. Retrospective taxation was
widely opposed, including by those within the ruling dispensation. Prime minister Manmohan Singh was
convinced that the proposed amendment in the IT Act would impact FDI inflows into the country, Mukherjee
wrote in his memoir, adding that Sonia Gandhi, Kapil Sibal, and P Chidambaram also expressed the apprehension
that the retrospective amendments would create a negative sentiment for FDI. When the Bhartiya Janata Party
came to power, it also continued with retrospective taxation; the Government did a somersault only when its
position became absolutely untenable in the international arena. It should not do anything that brings more
embarrassment to India.

112. The main idea of the passage is best represented by:


(a) The Government's decision to seek additional compensation from Union Carbide for the 1984 Bhopal gas
tragedy.
(b) The potential negative impact on foreign direct investment if the Government reopens the 1989 settlement
with Union Carbide.
(c) The unedifying episode of retrospective taxation and its impact on foreign investment in India.
(d) The Government's disregard for the ruling of the Supreme Court in the Vodafone case and its impact on
foreign investment in India.

113. Which of the following best describes the role played by the statement, "The Bhopal gas tragedy was caused by
a leak of methyl isocyanate gas from a Union Carbide pesticide plant in Bhopal, India in 1984, which killed over
3,000 people immediately and has since claimed thousands more lives due to related illnesses," in the context of
the passage?
(a) It supports the main idea that the Government's decision to seek additional compensation from Union Carbide
may have a negative impact on foreign direct investment.
(b) It challenges the main idea that retrospective taxation has a negative impact on foreign investment in India.
(c) It serves as background information to understand the situation described in the passage.
(d) It is not mentioned in the passage and therefore, does not play a role in the context of the passage.

114. Which of the following adds value to the information presented in the passage?
(a) The Indian Government's decision to seek additional compensation from Union Carbide will be beneficial
for the victims of the Bhopal gas tragedy and their families.
(b) The Indian Government's decision to seek additional compensation from Union Carbide will have no impact
on foreign direct investment in India.
(c) The Indian Government's decision to seek additional compensation from Union Carbide is a violation of the
principle of "sanctity of settlement" and will damage India's reputation in the international business
community.
(d) The Indian Government's decision to seek additional compensation from Union Carbide is motivated by
political considerations rather than a desire to provide fair compensation for the victims of the Bhopal gas
tragedy.

"N-5, 2nd Floor, Bajrang House, South Extension-1, New Delhi Pin Code: 110049" |+91-7676564400| https://www.toprankers.com Page 30 of 40
115. “The Indian Government's decision to seek additional compensation from Union Carbide will damage India's
reputation in the international business community?”
Which of the following, if true, would most strengthen the author's argument?
(a) Studies have shown that multinational corporations are more likely to invest in countries with a history of
honoring settlements and agreements.
(b) The Indian Government's decision to seek additional compensation from Union Carbide is a common
practice among other countries in similar situations.
(c) The Union Carbide corporation has publicly stated that it will continue to invest in India in the future even
if the Indian Government's decision to seek additional compensation is successful.
(d) The Indian Government has never disregarded previous settlements and agreements with multinational
corporations.

116. Which of the following is NOT mentioned as a reason for why the Indian Government's decision to seek
additional compensation from Union Carbide may have negative ramifications, as stated in the passage?
(a) Impact on foreign direct investment in India.
(b) The principle of "sanctity of settlement" would be affected.
(c) GoI has always successfully renegotiated settlements with MNCs.
(d) The Indian Government's move may bring more embarrassment to India.

117. What are the author's perceptions regarding the conduct of Pranab Mukherjee during his tenure as the Minister
of Finance?
(a) The author holds an affirmative attitude towards the actions and decisions taken by Pranab Mukherjee as the
Minister of Finance.
(b) The author maintains a indifferent stance towards the actions and decisions taken by Pranab Mukherjee as
the Minister of Finance.
(c) The author holds a critical view towards the decisions taken by Pranab Mukherjee as the Minister of Finance,
in regards to disregarding the ruling of the Supreme Court.
(d) The author does not express any views on the actions and decisions taken by Pranab Mukherjee as the
Minister of Finance within the passage.

Passage (Q.118-Q.123): Five-and-a-half years after she took office as the Prime Minister of New Zealand,
Jacinda Ardern has bucked the trend yet again by announcing her decision to step back from her role, not because
of the dip in her popularity but because months of deliberation has led her to the conclusion that she no longer
has what it takes to continue. As unexpected as her decision is, it is in keeping with Ardern’s image as a prime
minister who walks the talk. As the world’s youngest female head of government at the time of her election in
2017, Ardern was pitched headlong into the culture of machismo of her global counterparts — from former US
President Donald Trump to Brazilian President Jair Bolsonaro to UK Prime Minister Boris Johnson. But she
changed the narrative during her action-packed stint, carving a niche for her tiny nation by the strength of her
personality and “relentless positivity”.

She was a world leader and a mother who gave birth while in office and spoke at the United Nations general
assembly meeting with a three-month-old infant in tow. At a time of heightened Islamophobia, she handled the
2019 Christchurch mass shootings with sensitivity, respect, and most importantly, decisive action. She displayed
the same efficiency in dealing with a major earthquake as well as an unprecedented pandemic. In the
masculinisation of political discourse, Ardern’s leadership made possible on a global stage a realisation of grace,
efficiency and that other thing that she hopes will be her legacy — “someone who always tried to be kind”.

Does her decision to step back, then, seem like a fairy tale that has run out of steam? Given the post-pandemic
economic crisis, her party faces a difficult election in October. Once history’s dust has settled, however, Ardern’s
choice could turn out to be yet another pragmatic decision by a leader who knew when to stop, for herself and
for her nation.

"N-5, 2nd Floor, Bajrang House, South Extension-1, New Delhi Pin Code: 110049" |+91-7676564400| https://www.toprankers.com Page 31 of 40
118. Which of the following best captures the notion behind the author penning this passage?
(a) The author implied that Ardern's resignation is a demonstration of her inadequacy as a leader, due to her
inability to navigate the complex challenges of leading a nation in a post-pandemic economic crisis.
(b) The author posited that Ardern's resignation reflects her commitment to acting in the best interest of both
herself and the nation, exemplifying her pragmatic leadership style and her ability to recognize when it is
time to step down.
(c) The author argued that Ardern's resignation is an indication of her failure to maintain the support of her party
and electorate, thereby rendering her incapable of leading the nation effectively.
(d) The author contended that Ardern's resignation is a result of the masculinisation of political discourse and
her inability to compete with her global counterparts.

119. The information in the passage allows for which of the following conclusions to be drawn?
(a) Jacinda Ardern is currently the youngest woman prime minister of any country.
(b) Jacinda Ardern's leadership has been effective in dealing with the post-pandemic economic crisis in New
Zealand.
(c) Jacinda Ardern's decision to step down was influenced by the masculinization of political discourse and her
inability to compete with her global counterparts.
(d) None of the above.

120. Which of the following best describes the role of the statement "Jacinda Ardern's leadership has been recognized
as a role model for other women in politics" in the context of the passage?
(a) The statement neutralizes the passage's main idea that Ardern's decision to step down reflects her pragmatic
leadership style.
(b) The statement contradicts the passage's main idea that Ardern's decision to step down reflects her pragmatic
leadership style.
(c) The statement supports the idea that Ardern’s leadership made possible on a global stage a realisation of
grace, efficiency and that other thing that she hopes will be her legacy.
(d) The statement is erroneous.

121. Which of the following best describes how the passage portrays Jacinda Ardern's handling of the 2019
Christchurch mass shootings?
(a) The passage suggests that Ardern's handling of the situation was marked by insensitivity and a lack of
decisive action.
(b) The passage indicates that Ardern's handling of the situation was marked by a strong sense of compassion,
respect and resolution.
(c) The passage implies that Ardern's handling of the situation was marked by political expediency and lack of
empathy.
(d) The passage does not mention anything about the handling of the 2019 Christchurch mass shootings.

122. Which of the following is a fact from the passage rather than a claim?
(a) Jacinda Ardern was the world's youngest female head of government at the time of her election in 2017.
(b) Jacinda Ardern's leadership could be recognized as a role model by other women in politics.
(c) Jacinda Ardern's decision to step down was influenced by her inability to compete with her global
counterparts.
(d) The post-pandemic economic crisis in New Zealand was the core reason of her stepping down.

"N-5, 2nd Floor, Bajrang House, South Extension-1, New Delhi Pin Code: 110049" |+91-7676564400| https://www.toprankers.com Page 32 of 40
123. Which of the following statements is mentioned in the passage as something Jacinda Ardern hopes will be her
legacy?
(a) Jacinda Ardern's leadership has been recognized as a role model for other prime ministers in politics.
(b) Jacinda Ardern's decision to step down as Prime Minister of New Zealand reflects her pragmatic leadership
style.
(c) Jacinda Ardern's decision to step down was influenced by her inability to compete with her global
counterparts.
(d) Remembered as an individual who unswervingly upheld and exemplified the virtue of kindness in her tenure
as the Prime Minister.

Passage(Q.124-Q.129): Article 21A of the Constitution and the Right of Children to Free and Compulsory
Education Act, 2009 outline the fundamental right to education and the right to have free and compulsory
education for children aged 6-14 years. The Sarva Shiksha Abhiyan, which adopted a ‘zero rejection policy’,
emphasises that “every child with special needs, irrespective of the kind, category and degree of disability, is
provided meaningful and quality education”. India ratified the UN Convention on the Rights of Persons with
Disabilities. Aligning with this commitment, the government launched the Accessible India Campaign (Sugamya
Bharat Abhiyan) in 2015. An important pillar of this campaign is accessibility to the built environment. The
government has also been supportive of the principle of Leave No One Behind (LNOB), which is the central,
transformative promise of the 2030 Agenda for Sustainable Development. Anchored under the Mainstreaming
LNOB project by UN-Habitat, pilot training programmes on enhancing accessibility and inclusion were
implemented in two schools in Delhi with support from IIT Kharagpur and the Department of Social Welfare,
Delhi government.

Through interactive training sessions and simulation exercises that encourage empathy-building, more than 400
participants, including children, school faculty and staff, were informed about different types of disabilities
among children and the specific barriers experienced in different infrastructure zones of a school. After the
simulation exercises, children said they will make more efforts to assist their classmates whenever they can.
Various good practices, guidelines, and standards for making buildings, campuses, and infrastructure safe,
accessible, and inclusive were also showcased. Even if any combination of these accessibility features is added
or adapted in schools, it can go a long way in creating inclusive spaces.

Developing inclusive and accessible schools will be a big step towards not only challenging perceptions about
CWD, and the associated discrimination, but also in actualising the zero-rejection policy in schools.

A multi-pronged participatory approach towards providing an enabling environment for the empowerment of
future citizens is needed to ensure that stakeholders in the school ecosystem collectively work towards promoting
accessibility and inclusion in schools. This includes awareness and sensitisation programmes for children,
parents, and caregivers; training trainers for upskilling of school faculty and special educators and providing
access to updated teaching toolkits and materials; technical training for local government departments; and a co-
learning platform for knowledge-sharing between all.

124. Which of the following best represents the main purpose of the author in penning this passage?
(a) To advocate for a multi-pronged approach to promote accessibility and inclusion in schools for children with
disabilities.
(b) To inform about the different types of disabilities among children and the specific barriers experienced in
different infrastructure zones of a school.
(c) To demonstrate various good practices, guidelines, and standards for making buildings, campuses, and
infrastructure safe, accessible, and inclusive.
(d) To explain the zero-rejection policy in schools and how it is being implemented by the government.

"N-5, 2nd Floor, Bajrang House, South Extension-1, New Delhi Pin Code: 110049" |+91-7676564400| https://www.toprankers.com Page 33 of 40
125. Which of the following statements aligns most closely with the author's perspective on promoting accessibility
and inclusion in schools for children with disabilities?
(a) The government should not have a role in promoting accessibility and inclusion in schools; it is the
responsibility of individual schools and teachers.
(b) The best way to promote accessibility and inclusion in schools for children with disabilities is by
implementing policies and regulations that enforce compliance.
(c) Accessibility and inclusion can be achieved in schools for children with disabilities by providing specialized
training for teachers and staff only.
(d) A comprehensive and collaborative approach that involves multiple stakeholders, including government,
schools, teachers, and parents, is essential for promoting accessibility and inclusion in schools for children
with disabilities.

126. Which of the following, if true, would most strengthen the author's arguments in the passage for a comprehensive,
inclusive, and collaborative approach to promoting accessibility and inclusion in schools for children with
disabilities?
(a) Studies showing that strict punishment for non-compliance with accessibility regulations have been
successful in promoting accessibility and inclusion in schools.
(b) Data demonstrating that retrofitting existing school buildings to meet accessibility standards is sufficient for
promoting accessibility and inclusion in schools.
(c) Testimonies from educators and parents that specialized training for only a select group of teachers and
school staff is sufficient to promote accessibility and inclusion in schools.
(d) Research that demonstrates the high success rate of inclusive and collaborative approaches involving
multiple stakeholders in promoting accessibility and inclusion in schools for children with disabilities.

127. What is the main purpose of the last paragraph in the context of the passage?
(a) To provide an overview of the main features of the Accessible India Campaign.
(b) To discuss the importance of inclusive and accessible schools in actualizing the zero-rejection policy.
(c) To outline the steps needed to ensure stakeholders in the school ecosystem work towards promoting
accessibility and inclusion.
(d) To explain the benefits of the Mainstreaming LNOB project by UN-Habitat for children with disabilities.

128. Which of the following is a statement of fact from the passage rather than a statement of claim?
(a) Developing inclusive and accessible schools will be a big step towards challenging perceptions about CWD,
and the associated discrimination.
(b) The government has been supportive of the principle of Leave No One Behind (LNOB).
(c) The zero-rejection policy in schools is not important.
(d) All schools should have the same accessibility features.

129. According to a study, the number of students who failed their final exams increased by 30% this year. The school
board attributed the failure to the new teaching method that was implemented this year. Which of the following
can be inferred from the statement?
(a) The new teaching method was the primary cause of the increase in the number of students who failed their
final exams.
(b) The school board is responsible for the increase in the number of students who failed their final exams.
(c) The school board is not capable of handling the new teaching method.
(d) The students were not motivated to study this year.

"N-5, 2nd Floor, Bajrang House, South Extension-1, New Delhi Pin Code: 110049" |+91-7676564400| https://www.toprankers.com Page 34 of 40
Passage(Q.130-Q.135): During a visit to Colombo last week, the external affairs minister, S. Jaishankar,
announced India’s support for an International Monetary Fund debt restructuring plan for Sri Lanka. The
declaration was important in strengthening the hands of the Sri Lankan government in talks with the IMF, as the
island nation looks to rebuild a devastated economy, since India is among the country’s major creditors. But
New Delhi’s support for Colombo’s debt talks with the IMF also serves as an example of India’s proactive
approach in responding to the Sri Lankan crisis. That is welcome: far too often in the past, India has been slow
to react to difficult situations in its neighborhood. New Delhi was also quick to offer loans and aid to Sri Lanka
after its economy collapsed last year amid a sharp decline in foreign currency reserves and a spike in inflation.
Yet, New Delhi must be careful of not repeating another frequent folly: making promises it eventually fails to
keep.

At the heart of India’s response to Sri Lanka’s crisis is an unstated desire to be seen as the country’s primary
regional benefactor at a time when New Delhi’s biggest rival, Beijing, has appeared somewhat hesitant in its
financial support to Colombo’s efforts to kickstart the economy, once again. Yet India must ensure that its
approach to Sri Lanka — and to other neighbors — is fashioned fundamentally by its vision for the region and
New Delhi’s place in it, rather than by an external actor like China. That alone will make India a truly steady
partner in the eyes of its neighbors. Strategic gains over China will follow on their own. At the same time, India
must use the influence it gains from its support to the Sri Lankan economy to demand that Colombo moves
towards greater autonomy for the country’s Tamil-dominated Northern Province and that it respect Tamil rights
and sensitivities more broadly. Finally, India must do all of this without emboldening those who accuse it of
interfering in the internal politics of Sri Lanka. In Colombo, Mr Jaishankar met the former Sri Lankan presidents,
Mahinda Rajapaksa and Gotabaya Rajapaksa, whose family was overthrown from power last year amid a popular
people’s revolt. The Rajapaksas’ party remains in government, even though the family is largely unpopular in
Sri Lanka. How India navigates the maze of economic and political factors shaping Sri Lanka will decide whether
its current approach yields the results it seeks.

130. Which of the following is a brief summation of the passage? ?


(a) Sri Lanka is a ship navigating through a stormy sea.
(b) India's role in Sri Lanka is a chess game of strategy.
(c) Sri Lanka's economy is that of a bird trying to take off.
(d) India and Sri Lanka's relationship is a delicate dance.

131. Based on the information given in the passage, which of the following is an inference that can be drawn about
India-Sri Lanka ties?
(a) India is reluctant to support Sri Lanka's efforts to rebuild its economy.
(b) India's support for Sri Lanka is primarily driven by its desire to counteract China's influence in the region.
(c) India is not concerned about the autonomy of Tamil-dominated Northern Province and the rights of Tamils
in Sri Lanka.
(d) India is a steady partner for Sri Lanka and is committed to supporting its economic recovery while also
promoting greater autonomy for the country's Tamil-dominated Northern Province and respect for Tamil
rights.

132. Based on the information given in the passage, which of the following statements is true about the former Sri
Lankan presidents, Mahinda Rajapaksa and Gotabaya Rajapaksa?
(a) The meeting was held to discuss India's support for an International Monetary Fund debt restructuring plan
for Sri Lanka.
(b) The Rajapaksa family was overthrown from power last year amid a popular people's revolt and are currently
in opposition.
(c) The Rajapaksas' party is currently in government, even though the family is unpopular in Sri Lanka.
(d) The meeting was held to discuss India's demand for greater autonomy for the country's Tamil-dominated
Northern Province and respect for Tamil rights.

"N-5, 2nd Floor, Bajrang House, South Extension-1, New Delhi Pin Code: 110049" |+91-7676564400| https://www.toprankers.com Page 35 of 40
133. Which of the following is the author least likely to agree with?
(a) India's support for Sri Lanka is not influenced by its own vision for the region and its place in it.
(b) India should not be concerned about the autonomy of Tamil-dominated Northern Province and the rights of
Tamils in Sri Lanka.
(c) India's support for Sri Lanka should be conditioned on the country's respect for Tamil rights and autonomy
for the Tamil-dominated Northern Province.
(d) India's support for Sri Lanka's economic recovery is primarily driven by its unstated desire to counteract
China's influence in the region.

134. Based on the passage, what can be said to be true about the aid provided by a country to its neighbors?
(a) The aid should be unconditional and not linked to any political or internal matter of the neighbor country.
(b) The aid should be given primarily to counteract the influence of other countries in the region.
(c) The aid should be given based on the country's own vision for the region and its place in it, and linked to the
internal matters of the neighbor country
(d) The aid should be given only if the neighboring country is in dire need and without any consideration of the
country's vision for the region.

135. Which of the following, if true, would most weaken the author's arguments?
(a) India has provided aid to Sri Lanka in the past, but failed to follow through on the promises it made.
(b) India has repeatedly expressed its concern for the autonomy of Tamil-dominated Northern Province and the
rights of Tamils in Sri Lanka.
(c) India has openly stated that its support for Sri Lanka is primarily driven by its desire to counteract China's
influence in the region.
(d) India has a history of being slow to respond to difficult situations in its neighborhood.

"N-5, 2nd Floor, Bajrang House, South Extension-1, New Delhi Pin Code: 110049" |+91-7676564400| https://www.toprankers.com Page 36 of 40
SECTION - E: QUANTITATIVE TECHNIQUES

Direction (Q.136 – Q.139): Study the following information carefully and answer the related questions.
Female students in school A are 30% more than the male students in school B and sum of male students in
school B and C is 480. Total number of students in school A and C are in the ratio 8: 9 respectively and their
difference is 50. There are 170 and 100 female students in school C and B respectively.

136. How many students are there in school A, B and C together?


(a) 1050 (b) 1200 (c) 1100 (d) 1150

137. Total students in school A is what percentage more/less than the total students in school C
(a) 16.67% (b) 14.285% (c) 12.5% (d) 11.11%

138. What is the respective ratio of number of male students in school A and C?
(a) 1: 2 (b)4: 5 (c) 3: 4 (d)2: 3

139. Approximately what percent students in school B are males?


(a) 63% (b) 60% (c) 67% (d) 70%

Direction (Q.140 – Q.144): Read the following passage carefully and answer the given questions.
A shopkeeper sold three different Gun items viz.AK-47, AK-56 and AK-67 in three different month’s viz. April,
May and June.
I. Total AK-47 sold in April is same as total AK-67 sold in June and is also 20 more than total AK-47 and total
AK-67 sold in June and April respectively.
II. Total AK-47 sold in May is 20 less than the twice the total AK-47 sold in June and is also 20 more than total
AK-56 sold in May.
III. Total AK-56 sold in April and the total AK-67 sold in May is same, which is twice the total AK-67sold in
June.
IV. Total AK-56 sold in June is twice the total AK-47 sold in June and the total AK-47 sold all the three months
together is 400.

140. If the total AK-47 sold in April is 20% more than March which is 25% more than February, then find the total
number of AK-47 sold in February?
(a) 75 (b) 120 (c) 80 (d) 160

141. Total AK-47 sold in June is what percentage of the total AK-67 sold in May?
(a) 16.67% (b) 41.67% (c) 42.5% (d) 11.11%

142. What is the ratio between the total AK-56 and total AK-67 sold in all the given months together?
(a) 21: 23 (b) 30: 23 (c) 41: 27 (d) 33: 37

143. If total defective AK-47 sold in April, May and June is 20%, 25% and 30% respectively, then find the total non
defective AK-47 sold in all the given months?
(a) 452 (b) 253 (c) 519 (d) 301

144. Total AK-56 sold in June is what percentage more/less than the total AK-67 sold in May?
(a) 16.67% (b) 14.285% (c) 12.5% (d) 11.11%

"N-5, 2nd Floor, Bajrang House, South Extension-1, New Delhi Pin Code: 110049" |+91-7676564400| https://www.toprankers.com Page 37 of 40
Direction (Q.145 – Q.150): Study the following table carefully & answer the questions.
Table given below shows the percentage distribution of 5 items and ratio of each item sold by Flipkart and
Amazon during offer days in year 2018.

145. What is the difference of total item sold by Amazon of type A and by Flipkart of type D?
(a) 231 (b) 421 (c) 285 (d) 289

146. Item C sold by Flipkart is approximately what percent of item E sold by Amazon?
(a) 82% (b) 92% (c) 86% (d) 72%

147. What is average number of items sold of type A, C and E?


(a) 722 (b) None of these (c) 628 (d) 728

148. Find the ratio of items sold by Flipkart of type B to items sold by Amazon of type D?
(a) 3: 1 (b) 14: 3 (c) 7: 3 (d) 12: 5

149. Average of items sold by Amazon of type C and E is how much more or less than average of item sold by
Flipkart of type A and B?
(a) 11 (b) 7 (c) 8 (d) 10

150. Items sold of type C is what percent more or less than items sold of type A?
(a) 150% (b) 133% (c) 140% (d) 120%

"N-5, 2nd Floor, Bajrang House, South Extension-1, New Delhi Pin Code: 110049" |+91-7676564400| https://www.toprankers.com Page 38 of 40
Notes:-

"N-5, 2nd Floor, Bajrang House, South Extension-1, New Delhi Pin Code: 110049" |+91-7676564400| https://www.toprankers.com Page 39 of 40
Notes:-

"N-5, 2nd Floor, Bajrang House, South Extension-1, New Delhi Pin Code: 110049" |+91-7676564400| https://www.toprankers.com Page 40 of 40

You might also like